Logo Studenta

Solucionario-TeoradelaMedida-AndrsFelipeUribe

¡Este material tiene más páginas!

Vista previa del material en texto

See discussions, stats, and author profiles for this publication at: https://www.researchgate.net/publication/342672992
Solución a ejercicios seleccionados del libro Lebesgue Integration on Euclidean
Space
Research · July 2020
DOI: 10.13140/RG.2.2.10589.03046
CITATIONS
0
READS
130
1 author:
Some of the authors of this publication are also working on these related projects:
Sobre la independencia de la hipótesis del espacio normal de Moore View project
Andrés Felipe Uribe Zapata
National University of Colombia
6 PUBLICATIONS   8 CITATIONS   
SEE PROFILE
All content following this page was uploaded by Andrés Felipe Uribe Zapata on 04 July 2020.
The user has requested enhancement of the downloaded file.
https://www.researchgate.net/publication/342672992_Solucion_a_ejercicios_seleccionados_del_libro_Lebesgue_Integration_on_Euclidean_Space?enrichId=rgreq-d27a8684d2339d26ace5979d2589095e-XXX&enrichSource=Y292ZXJQYWdlOzM0MjY3Mjk5MjtBUzo5MDk0NjQ2MzQ5MjkxNTJAMTU5Mzg0NDY3NDU5OQ%3D%3D&el=1_x_2&_esc=publicationCoverPdf
https://www.researchgate.net/publication/342672992_Solucion_a_ejercicios_seleccionados_del_libro_Lebesgue_Integration_on_Euclidean_Space?enrichId=rgreq-d27a8684d2339d26ace5979d2589095e-XXX&enrichSource=Y292ZXJQYWdlOzM0MjY3Mjk5MjtBUzo5MDk0NjQ2MzQ5MjkxNTJAMTU5Mzg0NDY3NDU5OQ%3D%3D&el=1_x_3&_esc=publicationCoverPdf
https://www.researchgate.net/project/Sobre-la-independencia-de-la-hipotesis-del-espacio-normal-de-Moore?enrichId=rgreq-d27a8684d2339d26ace5979d2589095e-XXX&enrichSource=Y292ZXJQYWdlOzM0MjY3Mjk5MjtBUzo5MDk0NjQ2MzQ5MjkxNTJAMTU5Mzg0NDY3NDU5OQ%3D%3D&el=1_x_9&_esc=publicationCoverPdf
https://www.researchgate.net/?enrichId=rgreq-d27a8684d2339d26ace5979d2589095e-XXX&enrichSource=Y292ZXJQYWdlOzM0MjY3Mjk5MjtBUzo5MDk0NjQ2MzQ5MjkxNTJAMTU5Mzg0NDY3NDU5OQ%3D%3D&el=1_x_1&_esc=publicationCoverPdf
https://www.researchgate.net/profile/Andres-Uribe-Zapata?enrichId=rgreq-d27a8684d2339d26ace5979d2589095e-XXX&enrichSource=Y292ZXJQYWdlOzM0MjY3Mjk5MjtBUzo5MDk0NjQ2MzQ5MjkxNTJAMTU5Mzg0NDY3NDU5OQ%3D%3D&el=1_x_4&_esc=publicationCoverPdf
https://www.researchgate.net/profile/Andres-Uribe-Zapata?enrichId=rgreq-d27a8684d2339d26ace5979d2589095e-XXX&enrichSource=Y292ZXJQYWdlOzM0MjY3Mjk5MjtBUzo5MDk0NjQ2MzQ5MjkxNTJAMTU5Mzg0NDY3NDU5OQ%3D%3D&el=1_x_5&_esc=publicationCoverPdf
https://www.researchgate.net/institution/National_University_of_Colombia?enrichId=rgreq-d27a8684d2339d26ace5979d2589095e-XXX&enrichSource=Y292ZXJQYWdlOzM0MjY3Mjk5MjtBUzo5MDk0NjQ2MzQ5MjkxNTJAMTU5Mzg0NDY3NDU5OQ%3D%3D&el=1_x_6&_esc=publicationCoverPdf
https://www.researchgate.net/profile/Andres-Uribe-Zapata?enrichId=rgreq-d27a8684d2339d26ace5979d2589095e-XXX&enrichSource=Y292ZXJQYWdlOzM0MjY3Mjk5MjtBUzo5MDk0NjQ2MzQ5MjkxNTJAMTU5Mzg0NDY3NDU5OQ%3D%3D&el=1_x_7&_esc=publicationCoverPdf
https://www.researchgate.net/profile/Andres-Uribe-Zapata?enrichId=rgreq-d27a8684d2339d26ace5979d2589095e-XXX&enrichSource=Y292ZXJQYWdlOzM0MjY3Mjk5MjtBUzo5MDk0NjQ2MzQ5MjkxNTJAMTU5Mzg0NDY3NDU5OQ%3D%3D&el=1_x_10&_esc=publicationCoverPdf
Solución a ejercicios seleccionados del libro
Lebesgue Integration on Euclidean Space
Andrés Felipe Uribe Zapata
Universidad Nacional de Colombia Sede Medellín
Facultad de Ciencias
Escuela de Matemáticas
Medellín, Colombia
2020
Índice general
1. Introducción Rn 5
1.1. Conjuntos . . . . . . . . . . . . . . . . . . . . . . . . . . . . . . . . . . . . . . . 5
1.2. Conjuntos contables . . . . . . . . . . . . . . . . . . . . . . . . . . . . . . . . . 6
1.3. Topología . . . . . . . . . . . . . . . . . . . . . . . . . . . . . . . . . . . . . . . 7
1.4. Conjuntos compactos . . . . . . . . . . . . . . . . . . . . . . . . . . . . . . . . . 9
1.5. Continuidad . . . . . . . . . . . . . . . . . . . . . . . . . . . . . . . . . . . . . . 11
1.6. La función distancia . . . . . . . . . . . . . . . . . . . . . . . . . . . . . . . . . 13
2. La medida de Lebesgue en Rn 15
2.1. Construcción . . . . . . . . . . . . . . . . . . . . . . . . . . . . . . . . . . . . . 15
2.2. Propiedades de la medida de Lebesgue . . . . . . . . . . . . . . . . . . . . . . . 25
3. Invarianza de la medida de Lebesgue 35
3.1. Un poco de álgebra lineal . . . . . . . . . . . . . . . . . . . . . . . . . . . . . . 35
3.2. Matrices ortogonales . . . . . . . . . . . . . . . . . . . . . . . . . . . . . . . . . 40
3.3. Matrices generales . . . . . . . . . . . . . . . . . . . . . . . . . . . . . . . . . . 43
4. Algunos conjuntos interesantes 45
4.1. Un conjunto no medible . . . . . . . . . . . . . . . . . . . . . . . . . . . . . . . 45
4.2. La función de Lebesgue asociada con un conjunto de Cantor . . . . . . . . . . . 47
5. Álgebras de conjuntos y funciones medibles 50
5.1. Álgebras y σ-álgebras . . . . . . . . . . . . . . . . . . . . . . . . . . . . . . . . . 50
5.2. Conjuntos de Borel . . . . . . . . . . . . . . . . . . . . . . . . . . . . . . . . . . 56
5.3. Funciones medibles . . . . . . . . . . . . . . . . . . . . . . . . . . . . . . . . . . 57
5.4. Funciones simples . . . . . . . . . . . . . . . . . . . . . . . . . . . . . . . . . . . 61
6. Integración 64
6.1. Funciones no negativas . . . . . . . . . . . . . . . . . . . . . . . . . . . . . . . . 64
6.2. Funciones medibles en general . . . . . . . . . . . . . . . . . . . . . . . . . . . . 71
6.3. Casi en todas partes . . . . . . . . . . . . . . . . . . . . . . . . . . . . . . . . . 76
6.4. Integración sobre subconjuntos de Rn . . . . . . . . . . . . . . . . . . . . . . . . 77
6.5. Generalización: espacios de medida . . . . . . . . . . . . . . . . . . . . . . . . . 78
6.6. Algunos cálculos . . . . . . . . . . . . . . . . . . . . . . . . . . . . . . . . . . . 84
6.7. Miscelánea . . . . . . . . . . . . . . . . . . . . . . . . . . . . . . . . . . . . . . . 89
7. La integral de Lebesgue en Rn 90
7.1. La integral de Riemann . . . . . . . . . . . . . . . . . . . . . . . . . . . . . . . . 90
7.1.1. Semicontinuidad . . . . . . . . . . . . . . . . . . . . . . . . . . . . . . . 90
2
7.1.2. Integración de Riemann . . . . . . . . . . . . . . . . . . . . . . . . . . . 91
7.1.3. Contenido de Jordan . . . . . . . . . . . . . . . . . . . . . . . . . . . . . 97
7.2. Cambio lineal de variable . . . . . . . . . . . . . . . . . . . . . . . . . . . . . . . 101
7.2.1. Aproximación por funciones continuas . . . . . . . . . . . . . . . . . . . 101
7.2.2. Aproximación por funciones in�nitamente diferenciables . . . . . . . . . . 102
3
Introducción
1 Este documento es resultado de un curso dirigido de Introducción a la Teoría de la Medida
dictado por el profesor Diego Mejía Duque durante el intersemestral 2020-I en la Universidad
Nacional de Colombia, sede Medellín, en el cual se estudió el texto Lebesgue Integration on
Euclidean Space (véase [1]).
1Andrés Felipe Uribe Zapata, estudiante de Matemáticas de la Universidad Nacional de Colombia, sede
Medellín. Correo de contacto: anfuribeza@unal.edu.co.
4
Capítulo 1
Introducción Rn
1.1. Conjuntos
Problema 1. Sea {Ai}i∈I una colección de subconjuntos de Rn. Notemos que,
x ∈
(⋃
i∈I
Ai
)c
⇐⇒ x /∈
⋃
i∈I
Ai
⇐⇒ (∀i ∈ I)(x /∈ Ai)
⇐⇒ x ∈
⋂
i∈I
Aci ,
y en consecuencia, (⋃
i∈I
Ai
)c
=
⋂
i∈I
Aci .
Por otro lado, se tiene que,
x ∈
(⋂
i∈I
Ai
)c
⇐⇒ x /∈
⋂
i∈I
Ai
⇐⇒ (∃i ∈ I)(x /∈ Aj)
⇐⇒ x ∈
⋃
i∈I
Aci ,
y por consiguiente, (⋂
i∈I
Ai
)c
=
⋃
i∈I
Aci .
Problema 3. De�namos la siguiente sucesión decreciente de subconjuntos no vacíos de R:
(∀n ∈ N)
(
An :=
(
0,
1
n
))
,
y consideremos I :=
⋂
n∈NAn. Es claro que si x ≤ 0, entonces x no es un elemento de I. Ahora
bien, si x > 0, por la propiedad arquimediana, existe N ∈ N tal que x > 1/N y en consecuencia,
x /∈ AN . Más aún, note que para todo m ≥ N, x /∈ Am. Finalmente, tenemos que:⋂
n∈N
An = ∅.
5
1.2. Conjuntos contables
Problema 4. Sean A un conjunto y f : N→ A sobreyectiva. Para demostrar que A es contable,
de�namos la función g : A→ N tal que g(a) := mı́n{n ∈ N : f(n) = a}. Notemos que la función
está bien de�nida en virtud de la sobreyectividad de f, ya que para todo a ∈ A, el conjunto
{n ∈ N : f(n)= a} es no vacío. Es claro que g es un función inyectiva y por tanto, existe una
función biyectiva entre A y un subconjunto B de N. Si B es in�nito, entonces hay un biyección
entre A y N y si B es �nito, entonces A también lo es. Finalmente, en cualquier caso, A es un
conjunto contable.
Problema 5. Sean A un conjunto y f : A → N una función inyectiva. Notemos que el rango
de f, que llamaremos B, es un subconjunto de N. Si B es �nito, entonces inmediatamente A
es �nito. Por otro lado, si B es in�nito, entonces debe ser contable y en consecuencia, A es
contable. Observemos que si C ⊆ N es in�nito, no es difícil demostrar que C es contable, ya
que podemos de�nir, en virtud del teorema de la recursión , una biyección g : N → C tal
que g(0) := mı́n(C) y en general, para todo n ∈ N,
g(n) := mı́n(C \ {g(0), . . . , g(n− 1)}),
donde la buena de�nición de g está justi�cada por la in�nitud de C.
Problema 6. Sean A contable y B ⊆ A. Note que si A o B son �nitos, el resultado es inmediato,
así que supongamos que ambos son in�nitos y consideremos la función inclusión de B en A,
i : B → A. Ahora bien, como A es contable e in�nito, existe una función biyectiva f : A→ N.
Notemos que la composición f ◦ i : B → N es inmediatamente inyectiva y por tanto, por el
Problema 5, B es contable. Luego todo subconjunto de un conjunto contable, es contable.
Problema 7. Sea X un conjunto in�nito. En virtud del axioma de elección , �jemos una
función de elección f para el conjunto P(X)\{∅}. Ahora bien, por el teorema de la recursión ,
existe una función g : N→ X tal que:
a. g(0) = f(X).
b. (∀n ∈ N)(g(n) = f(X \ {g(1), . . . , g(n− 1)})).
Notemos que g está bien de�nida, ya que X es in�nito. Ahora, sea B := rango(g). Es inmediato
que B ⊆ X y como claramente g es inyectiva, resulta ser una biyección entre N y B, es decir, B es
un subconjunto contable de A. Finalmente, todo subconjunto in�nito contiene un subconjunto
contable.
Problema 8. Para demostrar que N × N es contable, basta con veri�car que la función f :
N× N→ N de�nida como:
f(n,m) := 2m−1(2n− 1)
es una función biyectiva, lo cual no es difícil. Sin embargo, notemos que este resultado se puede
generalizar: para todo k ∈ N, Nk es contable. En efecto, usemos el teorema fundamental de la
aritmética para demostrarlo. Fijemos k ∈ N y consideremos la función f : Nk → N de�nida
como sigue,
f(n1, . . . , nk) := p
n1
1 p
n2
2 · · · p
nk
k ,
6
donde pi representa el i-ésimo número primo. Es claro, por el teorema fundamental de la arit-
mética, que f es una función inyectiva y en consecuencia, por el Problema 5, concluimos que
Nk es contable.
Problema 9. Para demostrar que Q es contable, es común usar el razonamiento famoso que
hizo Cantor. Así que voy a hacer una demostración alternativa que se basa en las distintas
formas naturales en las que se puede de�nir el conjunto de los números racionales partiendo de
N. De�namos a Q ⊆ N3 como la colección de tripletas (i,m, n) que satisfacen:
a. m,n ≥ 1.
b. i ∈ {0, 1}.
c. mcd(m,n) = 1.
d. Si i = 0, entocnes n ≥ 2.
Esta es una forma natural y alternativa de de�nir a Q en donde es evidente que es contable,
pues es un subconjunto de un conjunto contable.
Problema 10. Consideremos una colección contable de conjuntos contables {An}n∈N. Así que,
como cada An es contable, en virtud del axioma de elección , podemos elegir una función
inyectiva fn : An → N. Ahora, de�namos la función f :
⋃
n∈NAn → N×N de�nida para x ∈ An
como f(x) := (n, fn(x)). Notemos que f inyectiva. En efecto, si x ∈ An, y ∈ Am y f(x) = f(y),
entonces (n, fn(x)) = (y, fm(y)), de donde se tiene que n = m y como particularmente fn es
inyectiva, se tiene que f es inyectiva. De manera que
⋃
n∈NAn se puede inyectar en N×N, el cual,
por el Problema 8 es contable, y en consecuencia, por el Problema 5, la unión es contable.
Problema 11. En el ejercicio se enumeró en intervalo (0, 1) como {x1, x2, · · · }, donde para
todo n ∈ N, se tiene que xn = 0.an1an2an3 · · · . Y se de�nió el número y = 0.b1b2b3 · · · donde
para todo n ∈ N, bn está de�nido de la siguiente manera:
bn :=

7 si ann = 1
1 si ann 6= 1
Es claro que y ∈ (0, 1) por la forma en que se de�nió su expansión decimal. Por otro lado,
supongamos que existe N ∈ N tal que y = xN y analicemos aNN es la expansión decimal de xN .
Por un lado, si aNN = 1, entonces BN = 7, en cuyo caso, aNN 6= bN y en consecuencia, y 6= XN .
Por el otro lado, si aNN 6= 1, entonces BN = 1, es decir, en cualquier caso las expansiones
decimales de y y xN di�eren en la N -ésima posición, implicando que y 6= xN . Finalmente, para
todo n ∈ N, y 6= xn. Notemos que este razonamiento muestra que R no es un conjunto contable,
pues se supuso que el intervalo (0, 1) se podía enumerar y se construyó un elemento que no se
logra capturar con dicha enumeración.
1.3. Topología
Problema 14. Sean x, y ∈ Rn y supongamos que x = (x1, . . . , xn) y que y = (y1, . . . yn).
a. Es inmediato por la de�nición.
7
b. Por un lado, si x = 0, entonces |x| =
√
0 = 0. Recíprocamente, si |x| = 0, entonces para
todo i ≤ n, se tiene que (xi − yi)2 = 0 y por tanto, para todo i ≤ n, xi = yi, es decir,
x = y.
c. d(x, y) =
√∑N
i=1(xi − yi)2 =
√∑N
i=1(yi − xi)2 = d(y, x).
d.
d(x, y) = |x− y| = |x− y + z − z| ≤= |x− z|+ |z − y| = d(x, z) + (z, y).
Problema 15. Sean x, y, z ∈ Rn. Por la desigualdad triangular tenemos que:
d(x, y) ≤ d(x, z) + d(y, z) y d(x, z) ≤ d(x, y) + d(y, z),
de donde se sigue que:
d(x, y)− d(x, z) ≤ d(y, z) y − d(y, z) ≤ d(x, y)− d(x, z),
es decir,
−d(y, z) ≤ d(x, y)− d(x, z) ≤ d(y, z),
y por propiedades de valor absoluto, �nalmente, obtenemos la desigualdad que buscábamos:
|d(x, y)− d(x, z)| ≤ d(y, z).
Problema 17. Supongamos que d(x, x′) ≤ r′ − r y sea y ∈ B(x, r). Así que d(x, y) < r y por
tanto,
d(x′, y) ≤ d(x′, x) + d(x, y) < r′ − r + r = r′,
luego, d(x′, y) < r′, es decir, y ∈ B(x′, r′) y en consecuencia,B(x, r) ⊆ b(x′, r′). Para el recíproco,
note que se sigue inmediatamente del Problema 16.
Problema 19. Consideremos, para todo n ∈ N, An :=
(−1
n
, 1
n
)
. Notemos que cada An es
abierto, sin embargo, ⋂
n∈N
An = {0},
el cual no es abierto. Luego, en general, la intersección de conjuntos abiertos, no es abierta.
Problema 20. Sean x ∈ Rn, r > 0 y y ∈ B(x, r). Así pues, d(x, y) < r, es decir, r−d(x, y) > 0
y por tanto, existe ry > 0 tal que ry < r − d(x, y). De donde se sigue que d(x, y) ≤ r − ry y
en virtud del Problema 17, tenemos que B(y, ry) ⊆ B(x, r). Luego, todo punto de B(x, r) es
interior y en consecuencia, es un conjunto abierto en Rn.
Problema 23. Sean A ⊆ Rn y x un punto límite de A. Razonando por contradicción, suponga-
mos que existe r > 0 tal que A∩B(x, r) es �nita y supongamos que A∩B(x, r) = {x1, . . . , XN},
donde para todo n ≤ N, xn 6= x. De�namos r1 := mı́n{d(x, xn) : n ≤ N}. Es claro, por de�ni-
ción de r1, que A ∩ B(x, r1) no contiene elementos, salvo x y en consecuencia, x no es punto
límite de A. En conclusión, pra todo r > 0, A ∩B(x, r) es in�nito.
Recíprocamente, si para todo r > 0, A∩B(x, r) es in�nita, entonces es claro, por de�nición,
que x es un punto límite de A.
8
Problema 25. Sean A ⊆ Rn y x ∈ A. Si x ∈ A, entonces para todo r > 0, x ∈ ApB(x, r), es
decir, para todo r > 0, A∩B(x, r) 6= ∅. Por otro lado, si x es un punto límite de A, el resultado
se sigue por la de�nición de punto límite.
Recíprocamente, supongamos que para r > 0, A ∩ B(x, r) 6= ∅ y supongamos que x no es
un punto límite de A. Así que, existe r1 > 0 tal que B(x, r1) \ {x} ⊆ Ac, sin embargo, como
particularmente A ∩B(x, r1) 6= ∅, entonces x ∈ A. Luego, x ∈ A.
Problema 26. Sean A ⊆ Rn y x ∈ A. Para todo n ∈ N, escojamos xn ∈ A tal que xn ∈
B(x, 1/n), note que esto es posible, ya que para todo r < 0, A ∩ B(x, r)∅, pues x ∈ A. Veamos
que ĺımn→∞ xn = x. En efecto, sea � > 0 y escojamos N ∈ N tal que 1/N < �. Sea n > N, así
que 1/n < 1/N, y calculemos:
d(x, xn) = |xn − x| <
1
n
<
1
N
< �.
Luego, efectivamente, la sucesión de punto de A, {xn}n∈N converge a x.
Recíprocamente, supongamos que existe una sucesiónde puntos de A {xn}n∈N que converge
a x y veamos que x ∈ A. Para ello, supongamos que x /∈ A y sea r < 0. Así que, existe N ∈ N
tal que N ≥ 1 y para todo n ≥ N, se tiene que d(xn, x) < r. Note que, como x /∈ A, entonces
para todo n > N, x 6= xn y xn ∈ B(x, r), es decir, A ∩ B(x, r) 6= ∅, con lo que x es un punto
límite de A.
Problema 30. ¾Es cierto en general que el interior de la clausura de un conjunto A es el mismo
conjunto? No, consideremos por ejemplo el conjunto A := [0, 1). Es claro que A = [0, 1] y por
tanto, A
◦
= (0, 1) 6= A. Luego, A◦ 6= A.
1.4. Conjuntos compactos
Problema 32.
a. Como el vacío está contenido en todos los conjuntos, el resultado se sigue trivialmente.
b. Sean A un conjunto �nito y {Gi}i∈I un cubrimiento abierto de A. Como A es �nito, existe
N ∈ N tal que A = {an : n ≤ N}. Por otro lado, ya que,
A ⊆
⋃
i∈I
Gi,
se sigue que para todo n ≤ N, existe un in tal que an ∈ Gin . De donde, �nalmente tenemos,
A ⊆
⋃
n≤N
Gin ,
es decir, Gi1 , . . . , GiN es un subcubrimiento �nito de A y en consecuencia, A es compacto.
c. Sean A y B conjuntos compactos y {Gi}i∈I un cubrimiento abierto de A∪B. De manera
que,
A ∪B ⊆
⋃
i∈I
Gi.
En consecuencia, {Gi}i∈I es un cubrimiento abierto tanto de A como de B y como ambos
son compactos, existen conjuntos de índices �nitos IA y IB tales que:
A ⊆
⋃
i∈IA
Gi y B ⊆
⋃
i∈IB
Gi,
9
de donde se sigue �nalmente que,
A ∪B ⊆
⋃
i∈IA∪IB
Gi,
y como IA ∪ IB es �nito, se sigue que A ∪B es compacto.
d. Notemos que, razonando por inducción sobre n ∈ N, donde n es la cantidad de conjuntos
compactos, se sigue inmediatamente del resultado anterior.
e. Sean x ∈ Rn y 0 < � < r. Veamos que B := B(x, r) no es compacto. Para ello, construya-
mos un cubrimiento abierto de B que no se puede reducir a uno �nito. Consideremos los
siguientes conjuntos abiertos:
(∀n ∈ N)
(
Gn := B
(
x, r − �
n
))
y de�namos G := {Gn}n∈N. Veamos que G es un cubrimiento abierto de B. En efecto, es
claro que es una colección de conjuntos abiertos y si d(x, y) < r, entonces, por la propiedad
arquimediana, existe k ∈ N tal que r− d(x, y) > �
k
, de donde se sigue que d(x, y) < r− �
k
,
es decir, y ∈ Gk y en consecuencia,
B(x, r) ⊆
⋃
n∈N
Gn.
Ahora bien, sean I ⊆ N �nito y M := máx(I). Si consideramos y ∈ Rn tal que:
r − �
M
< d(x, y) < r,
entonces y ∈ B, pero y /∈
⋃
i∈I Gi. Lo cual implica que ningún subconjunto �nito de
índices N es su�ciente para cubrir a B con elementos de G y por tanto, B no es compacto.
f. Similar al literal anterior, consideremos la siguiente sucesión de conjuntos abierto en Rn:
(∀n ∈ N)(Gn := B(0, n))
y la colección G := {Gn}n∈N . Es claro que G es un cubrimiento abierto de Rn. Ahora
bien, sean I ⊆ N �nito y M := máx(I). Tomemos y ∈ Rn tal que M < d(x, y). De esta
manera, y ∈ Rn, sin embargo, y /∈
⋃
n∈I Gn. Luego, ningún subconjunto �nito de índices
de N es su�ciente para cubrir a Rn con elementos de G. Finalmente, Rn no es un conjunto
compacto.
Problema 33. Sean A ⊆ Rn un conjunto compacto y de�namos, para todo n ∈ N los abiertos:
Gn := B(0, n).
Notemos que G := {Gn}n∈N es un cubrimiento abierto de A, ya que es un cubrimiento abierto
de Rn. Así que, por compacidad, existe un conjunto �nito de índices I ⊆ N tal que:
A ⊆
⋃
i∈I
Gi.
10
Como I es �nito, tiene un máximo elemento, digamos M := máx(I). Nótese que, por la cons-
trucción, si m < n, entonces Gm ⊆ Gn y en consecuencia, tenemos que:
A ⊆
⋃
i∈I
Gi ⊆ GM = B(0,M).
Finalmente, A ⊆ B(0,M) y por tanto, A es un conjunto acotado.
Problema 36. Sea {xn}n∈N una sucesión acotada de puntos en Rk. De�namos A como el rango
de la sucesión y consideremos dos posibles casos:
a. A es �nito: Así, existe x ∈ A tal que xn = x para in�nitos valores de n ∈ N. De manera
que si de�nimos el conjunto de índices I := {n ∈ N : xn = x}, tenemos que {xi}i∈I es una
subsucesión convergente de {xn}n∈N.
b. A es in�nito: Así, A es un subconjunto in�nito y acotado de Rk y en consecuencia, por el
teorema de Bolzano-Weierstrass, tiene un punto límite x. Construyamos un conjunto de
índices, I, inductivamente, de la siguiente manera: n1 := 1, y para todo j ∈ N, escojamos
nj+1 ∈ N, tal que nj+1 > nj y d(x, xnj+1) < 1/j. Así que, {xnj}j∈N, es una subsucesión
convergente, ya que si �jamos � > 0 y tomamos N ∈ N tal que 1/N < �, entonces para
todo j > N, se tiene que:
d(x, xnj+1) <
1
j
<
1
N
< �.
1.5. Continuidad
Problema 37. Sea f(x) := (f1(x), . . . fj(x)). Supongamos que f1, . . . , fj son funciones conti-
nuas en x0 ∈ Rj y sea � > 0. Así que, por la continuidad de cada una de la funciones, para todo
i ≤ j, existe δi > 0 tal que, si d(x, x0) < δ, entonces d(fi(x), fi(x0)) < �√j . Ahora, consideremos
δ := mı́n{δi : i ≤ j}. Supongamos, además que d(x, x0) < δ, entonces:
d(f(x), f(x0)) =
√√√√ j∑
i=1
(fi(x)− fi(x0))2
<
√
m
�2
m
=
√
�2
= �.
Y por tanto, f es continua en x0.
Recíprocamente, supongamos que f es continua en x0 y sea � > 0. Así que, existe δ > 0 tal
que, si d(x, x0) < δ, entonces:
d(f(x), f(x0)) =
√√√√ j∑
i=1
(fi(x)− fi(x0))2 < �,
de donde se sigue que, para todo i ≤ j, |fi(x) − fi(x0)| < �. Finalmente, para todo i ≤ j, se
tiene que fi es continua.
11
Problema 39. En el Problema 15 demostramos la siguiente desigualdad:
|d(x, y)− d(x, z)| ≤ d(y, z).
Veamos cómo lo podemos usar para demostrar que para x ∈ Rn �jo, la función f : Rn → R
de�nida por f(y) := d(x, y) es continua en Rn. Para ello, sea � > 0 y de�namos δ := �. Dados
y, z ∈ Rn tales que d(y, z) < δ, se sigue, por el Problema 15, que:
|f(y)− f(z)| = |d(x, y)− d(x, z)| ≤ d(z, y) < δ = �.
Finalmente, f es continua, de hecho, es uniformemente continua.
Problema 43. En general, las funiones continuas, en general, no envían abiertos en abiertos.
Por ejemplo, la función f : R→ R de�nida como f(x) := x2, envía el intervalo abierto (−π, π)
al conjunto [0, π2), el cual no es abierto.
Problema 44. De manera similar al ejercicio anterior, las funciones continuas, en general, no
envían conjuntos cerrados en conjuntos cerrados. Por ejemplo, consideremos la función f : R→
R de�nida por:
f(x) :=
1
x2 + 1
.
Note que f envía el conjunto cerrado R al conjunto (0, 1], el cual no es cerrado.
Problema 47. En contraste a los que ocurre con los conjuntos abiertos y cerrados, las funciones
continuas sí envían conjuntos compactos en conjuntos compactos; sin embargo, no devuelven
compactos en compactos, por ejemplo, consideremos la función f del Problema 44. La imagen
inversa bajo f del conjunto compacto [0, 1] es R, el cual no es compacto.
Problema 48. Consideremos la función f : R → R de�nida por f(x) := sin(x2). Es claro que
f es continua y acotada; sin embargo, no es uniformemente continua. Para verlo, sean � ∈ R tal
que 0 < � < 1 y δ > 0. Notemos que:
|f(
√
nπ)− f(
√
nπ + π/2)| = | sin(nπ)− sin(nπ + π/2)|
= | − sin(nπ + π/2)|
= | sin(nπ) cos(π/2) + sin(π/2) cos(nπ)|
= | cos(nπ)|
= | − 1|
= 1
> �.
Por otro lado, se tiene que:
|
√
nπ −
√
nπ + π/2| =
∣∣∣∣∣ π/2√nπ −√nπ + π/2
∣∣∣∣∣ < 1√n.
De modo que, en de�nitiva, si escogemos N > 1
δ2
, entonces |
√
Nπ −
√
Nπ + π/2| < δ. Final-
mente, concluimos que, si δ > 0, entonces podemos hallar x, y ∈ R tales que d(x, y) < δ y
d(f(x), f(y)) > �. Lo anterior muestra que f no es uniformemente continua en R.
12
Problema 49. Recordemos que una función f : A→ Rm satisface la condición de Lipschitz ,
si existe una constante C tal que para todo x, y ∈ A, se tiene que:
d(f(x), f(y)) ≤ Cd(x, y).
Supongamos que f satisface dicha condición y veamos que es uniformemente continua. Sea � > 0
y de�namos δ := �/C. Ahora, sean x, y ∈ A tal que d(x, y) < δ, es decir, d(x, y) < �/C y en
consecuencia, Cd(x, y) < �. Finalmente, tenemos que:
d(f(x), f(y)) ≤ Cd(x, y) < �,
mostrando así que f es, efectivamente, uniformemente continua.
1.6. La función distancia
Problema 50. Sean A ⊆ Rn y x ∈ Rn. Por un lado, supongamos que d(x,A) = 0. Entonces,
podemos hallar y ∈ A tal que d(x,A) = d(x, y), con lo que d(x, y) = 0 y por tanto, x = y ∈ A.
Recíprocamente, supongamos que x ∈ A. Por el Problema39, existe una sucesión de puntos
de A, {xn}n∈N, que converge a x y por consiguiente,
ĺım
n→∞
d(xn, x) = 0.
De manera que,
d(x,A) = ı́nf{d(x, y) : y ∈ A} ≤ ı́nf{d(xn, x) : n ∈ N} = 0,
es decir, d(x,A) ≤ 0. Finalmente, ya que para todo y ∈ A, d(x, y) ≤ 0, tenemos que d(x,A) ≥ 0
y por consiguiente, d(x,A) = 0.
Problema 51. Sean A y B subconjuntos de Rn tales que ∅ 6= A ⊆ B y �jemos x ∈ Rn. Así
pues, tenemos que:
{d(x, y) : y ∈ A} ⊆ {d(x, y) : y ∈ B},
y en consecuencia,
ı́nf{d(x, y) : y ∈ B} ≤ ı́nf{d(x, y) : y ∈ A}.
De manera que,
d(x,B) = ı́nf{d(x, y) : y ∈ B}
≤ ı́nf{d(x, y) : y ∈ A}
= d(x,A).
Y �nalmente, d(x,B) ≤ d(x,A).
Problema 54. Sean F cerrado y G abierto tales que F ⊆ G ⊆ Rn. Decimos que una función
continua f : Rn → R es una función de Urysohn , si para todo x ∈ Rn, 0 ≤ f(x) ≤ 1, para
x ∈ F, f(x) = 1 y para x ∈ Gc, f(x) = 0. Usemos la existencia de funciones de Urysonh para
demostrar el lema de Urysohn. Sean F1 y F2 subconjuntos abiertos y disjuntos de Rn. Así
13
pues, F c2 es abierto y F1 ⊆ F c2 . Sea f una función de Urysohn para el par F1 ⊆ F c2 y consideremos
los conjuntos:
G1 := {x ∈ Rn : f(x) > 1/2} y G2 := {x ∈ Rn : f(x) < 1/2}.
Como f es una función de Urysohn, si x ∈ F1, entonces f(x) = 1 y en consecuencia, x ∈ G1.
Y por otro lado, si x ∈ F2, entonces f(x) = 0, por lo cual, x ∈ G2. De manera que F1 ⊆ G1,
F2 ⊆ G2, es claro además que G1 y G2 son disjuntos y G1 y G2 son abiertos, ya que ambos
conjuntos son imágenes inversas bajo f de conjuntos abiertos.
Problema 55. Supongamos que A = {a}. Así pues, calculando el diámetro de A, tenemos que:
diam(A) = sup{d(x, y) : x, y ∈ A} = sup{d(a, a)} = 0.
Recíprocamente, supongamos que diam(A) = 0.Así pues, como para todo x, y ∈ A, d(x, y) ≥
0, tenemos que, para todo x, y ∈ A, d(x, y) = 0, es decir, para todo x, y ∈ A, se sigue que x = y,
con lo cual, A consiste de un único punto.
Problema 56. Sea A ⊆ Rn un conjunto acotado. Así pues, existe z ∈ Rn y r > 0, tal que
A ⊆ B(z, r). Sean x, y ∈ A y calculemos d(x, y) :
d(x, y) ≤ d(x, z) + d(z, y) < r + r = 2r.
Razón por la cual, el conjunto {d(x, y) : x, y ∈ A} está acotado superiormente y por tanto,
diam(a) <∞.
Recíprocamente, si diam(A) <∞, entonces si �jamos a ∈ A, para todo x ∈ A, se tiene que
d(a, x) ≤ diam(A), de donde se sigue �nalmente que A ⊆ B(a, diam(A) + 1), es decir, A es
acotado.
14
Capítulo 2
La medida de Lebesgue en Rn
2.1. Construcción
Problema 1. Empecemos por de�nir lo que es un subespacio afín de Rn. Un subespacio afín
de Rn es un conjunto de la forma {x0 + x : x ∈ E} donde x0 ∈ Rn es �jo y E es un subespacio
del R-espacio vectorial Rn.
Sea I ⊆ Rn un rectángulo especial, digamos I = [a1, b1], · · · , [an, bn].
a. Veamos que (a) =⇒ (c). Supongamos que λ(I) 6= 0 y veamos que I está contenido en
un subespacio afín de Rn con dimensión menor que n. Como λ(I) 6= 0, existe i ∈ Jn, tal
que ai = bi. Así que, consideremos el conjunto de punto de Rn cuya i-ésima coordenada
es 0, es decir:
E := {x ∈ Rn : xi = 0}.
Notemos que E es un subespacio vectorial de Rn. En efecto, es inmediato que 0 ∈ E y
si x, y ∈ E y r, s ∈ R, entonces la i-ésima componente de los vectores rx y sy es cero y
en consecuencia, rx + sy ∈ E; es decir, E es un subespacio vectorial de Rn. Ahora bien,
�jemos x0 ∈ I y sea y ∈ I. Si de�nimos x := y − x0, entonces la i-ésima coordenada de x
es cero, ya que la i-ésima coordenada de x0 y de y coinciden, es decir, x ∈ E. De manera
que, para todo y ∈ I, existe ∈ E tal que y = x0 + x y por tanto, I es un subconjunto del
espacio afín {x0 + x : x ∈ E}. Por otro lado, notemos que dim(E) = n − 1, ya que una
base para E consiste, por ejemplo, en la base canónica para Rn sin considerar el vector
canónico ei. Finalmente, I es un subconjunto de un subespacio afín de Rn cuya dimensión
es estrictamente menor que n.
b. Veamos que (b) =⇒ (a). Supongamos que λ(I) 6= 0 y veamos que I◦ 6= ∅. Ya que
λ(I) 6= 0, para todo i ∈ Jn, se tiene que ai < bi. Consideremos x como el punto medio de
todos los intervalos que componen a I, es decir:
x :=
(
a1 + b1
2
, . . . ,
an + bn
2
)
∈ Rn,
y consideremos la longitud de los intervalos que componen a I:
r := mı́n
{
b1 − a1
2
, . . . ,
bn − an
2
}
> 0.
El punto x es nuestro candidato a ser un punto interior de I. Para verlo, sea y ∈ B(x, r).
Así que |y − x| < r y en consecuencia, para todo i ∈ Jn, tenemos que |yi − xi| < r. Sea
15
i ∈ Jn. Por lo anterior, |yi − xi| < r. De manera que:∣∣∣∣yi − (ai + bi)2
∣∣∣∣ < bi − ai2 =⇒ ai − bi2 < yi − ai + bi2 < bi − ai2
=⇒ ai − bi < 2yi − ai − bi < bi − ai
=⇒ 2ai < 2yi < 2bi
=⇒ ai < yi < bi.
Es decir, yi ∈ [ai, bi] y en consecuencia, y ∈ I. Luego, B(x, r) ⊆ I y por tanto, x es,
efectivamente, un punto interior de I, de donde se sigue que I◦ 6= ∅. Finalmente, si
I◦ = ∅, entonces λ(I) = 0.
Problema 2. En el plano R2, consideremos el rectángulo especial I := [0, 1] × [0, 1]. Parti-
cionemos a I en nueve rectángulos que no se solapan de longitud 1
3
en ambas coordenadas y
consideremos:
G1 :=
[
1
3
,
2
3
]
×
[
1
3
,
2
3
]
,
es decir, el cuadrado central. Ahora, consideremos I \ G1, es decir, la colección de rectángulos
especiales como se indica en la siguiente �gura:
Figura 2.1: I \G1
Notemos además que λ(I \ G1) = 89 . Ahora tomemos los 8 rectángulos restantes y hagamos
el mismo proceso de dividir en 9 rectángulos y sea G2 la unión de los cuadrados centrales
resultantes. Entonces I \G1 \G2 es un polígono especial representado como 64 cuadrados que
no se traslapan y además λ(I \G1 \G2) = 6481 . Entonces calculemos la medida de Lebesgue del
polígono especial I \
⋃N
i=1Gi. Notemos que después de N ∈ N pasos, la medida de Lebesgue de
cada rectángulo es 1
9N
. Por otro lado, en el paso N al quitar los cuadrados centrales, quedan 8N
ractángulos. De manera que, en el N -ésimo paso,
λ
(
I \
N⋃
i=1
Gi
)
=
(
8
9
)N
.
16
Problema 3. Sean G ⊆ Rn abierto y P ⊆ G un polígono especial. Notemos que G \ P es no
vacío, ya que si fuese vacío, entonces P = G y en consecuencia, P sería cerrado por ser polígono
especial, y abierto. Sin embargo, esto implica que P = Rn o bien G = ∅, lo cual contradice
que P sea un polígono especial. De manera que, efectivamente, G \ P es no vacío. Ahora bien,
ya que G es abierto y P es cerrado, se sigue que G \ P es abierto y ya vimos que es no vacío.
De modo que, podemos hallar un polígono especial P1 ⊆ G \ P tal que P ◦1 6= ∅. Consideremos
P ′ := P ∪ P1. Así pues, es claro que P ⊆ P ′ ⊆ G. Por otro lado, notemos que P y P1 no
se traslapan, más aún, son disjuntos y por consiguiente, por las propiedades de la medida de
Lebesgue para rectángulos especiales y el Problema 1, tenemos que:
λ(P ) < λ(P1) + λ(P ) = λ(P1 ∪ P ) = λ(P ′),
es decir, λ(P ) < λ(P ′).
Problema 4.
a. Supongamos que G ⊆ Rn es abierto y acotado. Veamos que la medida de Lebesgue de G
es �nita, es decir que λ(G) < ∞. Como G es acotado, existen x ∈ Rn y r > 0 tales que
G ⊆ B(x, r). Sin embargo, podemos hallar un rectángulo especial I que contiene a B(x, r)
de la siguiente manera:
I := [x1 − r, x1 + r]× · · · × [xn − r, xn + r].
Veamos que, en efecto, B(x, r) ⊆ I. Sea y ∈ B(x, y). Así que, |x−y| < r y en consecuencia,
para i ∈ Jn, se tiene que |xi − yi| < r y por tanto, yi ∈ [xi − r, xi + r], es decir, y ∈ I y
por tanto, B(x, r) ⊆ I. Ahora bien, sea P ⊆ G un polígono especial. Así pues, tenemos
que, P ⊆ G ⊆ B(x, r) ⊆ I, luego, P ⊆ I y como P e I son polígonos especiales, por las
propiedades, tenemos que λ(P ) ≤ λ(I), es decir, λ(I) es una cota superior del conjunto
{λ(P ) : P ⊆ G y P es un polígono especial}. Razón por la cual,
λ(G) = sup{λ(P ) : P ⊆ G y P es un polígono especial} ≤ λ(I) <∞.
Así que, �nalmente, λ(G) < ∞. Sin embargo, podemos decir un poco más, ya que por
de�nición, la medida de I es:
λ(I) = (x1 + r − x1 + r) · · · (xn + r − xn + r) = (2r) · · · (2r) = (2r)n,
podemos concluir que no solo la medida de G es �nita, si no que está acotada por el
número (2r)n, es decir, λ(G) ≤ (2r)n.
b. En el plano R2, de�namos el conjunto abierto: G :=
{
(x, y)∈ R2 : 1 < x y 0 < y < 1
x
}
.
Y veamos que λ(G) =∞. De�namos, para todo n ∈ N, los siguientes polígonos especiales:
In := [3
n−1, 3n]×
[
1
3n+3
,
1
3n+2
]
.
Notemos que, para todo n > 1, In es un subconjunto de G. En efecto, sean n > 1 y
(x, y) ∈ In, entonces x ∈ [3n−1, 3n], es decir, x ≥ 3n−1 > 1 y, por otro lado, y ∈
[
1
3n+3
, 1
3n+2
]
,
por tanto, como x ∈ [3n−1, 3n], tenemos que:
0 <
1
3n+3
≤ y ≤ 1
3n+2
<
1
x
,
17
Figura 2.2: Idea visual de G.
es decir, (x, y) ∈ G y por consiguiente, para todo n ∈ N, In ⊆ G. Además, no es difícil
demostrar que dichos rectángulos no se solapan.
Ahora bien, por de�nición, podemos calcular la medida de cada In:
λ(In) = (3
n − 3n−1)
(
1
3n+2
− 1
3n+3
)
=
3n
3n+2
− 3
n
3n+3
− 3
n−1
3n+2
+
3n−1
3n+3
=
1
9
− 1
27
− 1
27
+
1
81
=
4
81
.
Así que, para todo n ∈ N, λ(IN) = 481 . Fijemos M ∈ R tal que M > 0 y sea N ∈ N tal
que N > 81
4
M +1. De�namos el polígono especial P :=
⋃N
k=2 Ik ⊆ G. Como no se solapan,
tenemos que:
λ(P ) =
N∑
k=2
λ(Ik) =
4
81
(N − 1) > M.
De modo que, el conjunto {λ(P ) : P ⊆ G y P es un polígono especial} no es acotado y
en consecuencia, λ(G) =∞.
Problema 5. Sea {Gi}i∈I una colección de subconjuntos abiertos y disjuntos de Rn. Conside-
remos los conjuntos de dicha colección que no son vacíos:
J := {i ∈ I : Gi 6= ∅},
y veamos que J es contable. Para ello, sea i ∈ J. Como Gi es abierto y no vacío, existe un
rectángulo especial P ⊆ Gi y digamos que P = [a1, b1]× · · · × [an, bn]. Por la densidad de Q en
R, para todo k ∈ Jn, existe qik ∈ Q tal que ak ≤ qik ≤ bk. Así que,
qi = (q
i
1, . . . , q
i
n) ∈ P ⊆ Gi.
18
Es decir, para todo i ∈ J, existe qi ∈ Qn tal que qi ∈ Gi. Ahora bien, usando el Problema
8, fácilmente podemos demostrar que Qn es contable y en consecuencia, existe una función
biyectiva g : Qn → N. Consideremos ahora, la función f : J → Qn de�nida como sigue:
f(i) := qi ∈ Gi.
Notemos que, como la colección {Gi}i∈I es disjunta, entonces f es inyectiva. Para concluir,
de�namos la función composición h : J → N tal que h(i) := g(f(i)). Ya que f y g son funciones
inyectivas, h es una función inyectiva y por consiguiente, por el Problema 5, J es contable.
Finalmente, solo una cantidad contable de conjuntos de {Gi}i∈I es no vacío.
Problema 7. Supongamos que {(ak, bk)}k∈I⊆N es una colección de intervalos de R disjuntos.
Así pues, como cada intervalo (ak, bk) es abierto, por las propiedades de la medida de Lebesgue
para conjuntos abiertos, tenemos que:
λ
(⋃
k
(ak, bk)
)
=
∑
k
λ(ak, bk) =
∑
k
(bk − ak).
De donde se sigue que, efectivamente,
λ
(⋃
k
(ak, bk)
)
=
∑
k
(bk − ak).
Problema 8. Es un ejercicio básico de análisis demostrar que las bolas abiertas y cerradas de
Rn son arco conexas y en consecuencia conexas. De manera que, en particular B(0, 1) ⊆ R2 es
conexa. Ahora bien, razonando por contradicción, supongamos que,
B(0, 1) =
⋃
i∈I
Ci,
donde cada Ci es un rectángulo abierto no vacío y la unión es disjunta. Notemos que I debe
contener más de un elemento, ya que es inmediato que B(0, 1) no es un rectángulo abierto. De
modo que, I tiene por lo menos dos elementos y por consiguiente, podemos descomponer a I
como I = I1∪I2 donde I1∩I2 = ∅. Así que, también podemos descomponer B(0, 1) como sigue:
B(0, 1) =
⋃
i∈I1
Ci ∪
⋃
i∈I2
Ci.
No obstante,
⋃
i∈I1 Ci y
⋃
i∈I2 son conjuntos abiertos y no vacíos, lo cual contradice la conexidad
de B(0, 1). Finalmente, B(0, 1) no se puede descomponer como unión disjunta de rectángulos
abiertos.
Problema 10. Consideremos el siguiente subconjunto de Rn:
G =
∞⋃
k=1
Ik,
donde cada Ik es un rectángulo especial y además dichos rectángulos no se solapan. Calculemos
la medida de G. Por un lado, tenemos que:
λ(G) = λ
(
∞⋃
k=1
Ik
)
≤
∞∑
k=1
λ(Ik).
19
Por el otro lado, tenemos que:
∞∑
k=1
λ(Ik) =
∞∑
k=1
λ(I◦k) = λ
(
∞⋃
k=1
I◦k
)
≤ λ(G),
donde, la primera igualdad se da por la propiedad (O7), la segunda usando la propiedad (O6),
teniendo en cuenta que los interiores de los rectángulos especiales que no se solapan son disjuntos
y, la tercera igualdad se da por la propiedad (O4), considerando que la unión de abiertos es
abierta. Finalmente, tenemos que:
λ(G) =
∞∑
k=1
λ(Ik).
Problema 11. Recordemos que, en el Problema 2, Gn se componía de la unión de los cuadrados
centrales que se eliminaban en el paso n ∈ N. Así que, en el N -ésimo paso, tenemos que GN es
la unión de 8N−1 cuadrados que además tienen medida 1
9N
. De manera que, como son disjuntos,
tenemos que:
(∀n ∈ N)
(
λ(Gn) =
8n−1
9n
)
.
Ahora bien, como los conjuntosGn son disjuntos, podemos usar la propiedad (O6) para calcular:
λ
(
∞⋃
k=1
Gk
)
=
∞∑
k=1
λ(Gk)
=
∞∑
k=1
8k−1
9k
=
∞∑
n=0
8n−1
9n
− 1
8
=
1
8
∞∑
n=0
(
8
9
)n
− 1
8
=
9
8
− 1
8
= 1.
De manera que, �nalmente:
λ
(
∞⋃
k=1
Gk
)
= 1.
Problema 12. Sabemos, por el Problema 9 del Capítulo 1, que los números racionales son
contables y por tanto, los podemos enumerar de la siguiente manera: Q = {qk}k∈N. Ahora bien,
sea � > 0 y de�namos, para todo k ∈ N el intervalo abierto:
Gk :=
(
qk −
�
2k+2
, qk +
�
2k+2
)
⊆ R.
20
Notemos que, para todo k ∈ N, se tiene que qk ∈ Gk y por tanto,
Q ⊆ G :=
∞⋃
k=1
Gk.
Además, G es abierto y podemos calcular la medida de cada Gk de la siguiente manera:
λ(Gk) = qk +
�
2k+2
− qk +
�
2k+2
= 2
�
2k+2
=
�
2k+1
.
De manera que, como cada Gk es abierto, podemos usar la propiedad (O5) para calcular:
λ(G) = λ
(
∞⋃
k=1
Gn
)
≤
∞∑
k=1
λ(Gk) =
∞∑
k=1
�
2k+1
=
�
2
∞∑
k=1
1
2k
=
�
2
< �.
Finalmente, Q ⊆ G ⊆ R y λ(G) < �.
Observación 1. Si bien en este punto no hemos de�nido lo que signi�ca la medida para un
subconjunto arbitrario de R, una vez se haya de�nido, se puede generalizar de manera inmediata
el razonamiento del ejercicio anterior para demostrar que todo subconjunto contable de R tiene
medida cero. Incluso, se podría modi�car para Rn.
Problema 13. Razonando por contradicción, supongamos que R es contable. Así que, lo po-
demos enumerar de la forma R = {rk}k∈N. Notemos que si reemplazamos qk por rk en la
demostración anterior, todo funciona exactamente igual y en consecuencia, tendríamos que pa-
ra todo � > 0, λ(R) ≥ 0, de donde, λ(R) = 0, lo cual es una contradicción, pues sabemos, por
la propiedad (O3), que λ(R) =∞.
Problema 14. Sea x ∈ [0, 1]. Supongamos que x ∈ C. Entonces debemos construir una expan-
sión ternaria para x que solo contenga 0′s y 2′s. Para ello, continuemos con la idea que indica
el libro. Ya que x ∈ Gc1, entonces x ∈ [0, 13 ]∪ [
1
3
, 1]. Si x ∈ [0, 1
3
], de�nimos α1 := 0 y si x ∈ [23 , 1],
de�nimos α1 := 2. De manera similar, x ∈ Gc2, es decir,
x ∈
([
0,
1
9
]
∪
[
2
9
,
1
3
])
∪
([
2
3
,
7
9
]
∪
[
8
9
, 1
])
y entonces, si x ∈ [0, 1
9
]∪ [2
9
, 1
3
], de�nimos α2 := 0 y si x ∈ [23 ,
7
9
]∪ [8
9
, 1], de�nimos α2 := 2. Así, en
general, podemos construir una sucesión {αn}n∈N de 0‘s y 2′s. Notemos que, por la construcción
de C, es claro que:
(∀n ∈ N)
(
|x− .α1α2 · · ·αn| <
1
3n
)
,
de manera que, la expansión ternaria que acabamos de de�nir, converge a x.
Recíprocamente, supongamos que x tiene una expansión ternaria 0.α1α2 . . . , que solo con-
siste de 0‘s y 2‘s. Ya que x ∈ Gn si, y solo si, αn = 1 en todas las expansiones ternarias de x,
entonces para todo n ∈ N, x /∈ Gn, es decir, x ∈ C.
Problema 16. Veamos que el conjunto de Cantor contiene puntos distintos a los extremos de
los intervalos que se extraen, es decir, puntos distintos de los extremos de los intervalos que
componen a cada Gn para n ∈ N. Empecemos por notar que los extremos de los intervalos
componentes de cada Gn son de la forma
j
3k
para j, k ∈ N.
21
Ahora bien, la expansión ternaria de 1
8
es ,02020202 . . . y además, por ser una serie geomé-
trica, tenemos que:
1
4
= 2
∞∑
n=1
1
9n
=
1
8
= ,0202020202 . . . ,
así que, por Problema 14, 1
4
∈ C, y que su expansión decimal ternaria solo está compuesta de
0′s y 2′s. Notemos que, 1
4
no puede tener la forma j
3k
, ya que si así fuera, entonces3k = 4j
y por tanto, 4 ÷ 3k, lo cual es absurdo y se veri�ca fácilmente por inducción, usando el Lema
de Euclides. Finalmente, el número 1
4
∈ C y no es el extremo de uno de los intervalos que se
eliminan en cada paso en la construcción de C.
De manera completamente análoga, se puede comprobar que
1
10
= 8
∞∑
n=1
1
81n
= ,002200220022 · · · ∈ C,
y que 1
10
no es el extremo de un intervalo que se elimina en la construcción de C.
Problema 17. Supongamos que x ∈ C. Así que, por la caracterización del Problema 14, tene-
mos que:
x =
∞∑
n=1
αn
3n
,
donde para todo n ∈ N, se tiene que αn ∈ {0, 2}. Como
∑∞
n=1
1
3n
= 1, entonces tenemos que:
1− x =
∞∑
n=1
2
3n
−
∞∑
n=1
αn
3n
=
∞∑
n=1
2− αn
3n
,
y como, αn ∈ {0, 2}, entonces 2 − αj ∈ {0, 2}, es decir, la expansión ternaria de 1 − x está
compuesta solo de 0′s y 2′s y en consecuencia, 1− x ∈ C.
Recíprocamente, si 1− x ∈ C, entonces:
1− x =
∞∑
n=1
αn
3n
,
de donde se sigue que:
x = 1−
∞∑
n=1
αn
3n
=
∞∑
n=1
2
3n
−
∞∑
n=1
αn
3n
=
∞∑
n=1
2− αn
3n
,
y como para todo n ∈ N, se tiene que αn ∈ {0, 2}, entonces 2− αn ∈ {0, 2}, es decir, x ∈ C.
Problema 18.
a. 1
252
: notemos que la expansión ternaria de 1/252, es:
1
252
= ,00000222000222000222000222 . . . ,
y por tanto, 1
252
∈ C.
22
b. 31
121
: notemos que la expansión ternaria de 31/121 es:
31
121
= ,020220202202022 . . . ,
y por tanto, 31
121
∈ C.
Problema 20. Sea x ∈ C tal que x y 2x−1 están en C. Así pues, por el Problema 17, tenemos
que 1− x ∈ y además,
2x− 1 ∈ C =⇒ 1− (2x− 1) ∈ C
=⇒ 1− 2x+ 1 ∈ C
=⇒ 2− 2x+ 1 ∈ C
=⇒ 2− 2x ∈ C
=⇒ 2(1− x) ∈ C.
Luego, 1 − x, 2(1 − x) ∈ C. Ahora bien, supongamos que los números x ∈ [0, 1] tales que
x, 2x ∈ C son de la forma {δi}i∈I (estos números se caracterizan en el Problema 19 ). Entonces
como 1− x, 2(1− x) ∈ C, entonces 1− x = δi y por tanto, x = 1− δi para algún i ∈ I.
Problema 21. Sea z ∈ [0, 2] y construyamos x, y ∈ C tales que z = x + y. Como lo indica el
libro, consideremos z
2
∈ [0, 1]. Así pues, z
2
tiene una expansión ternaria, digamos,
z
2
= .γ1γ2γ3 . . . ,
y de�namos, para todo n ∈ N los siguientes números αn, βn tales que γn = αn+βn2 :
αn :=

0 si γn = 0.
2 si γn = 1 ó γn = 2.
y además,
βn :=

0 si γn = 0 ó γn = 1.
2 si γn = 2.
Por la construcción, es claro que x := .α1α2α3 · · · ∈ C y x := .β1β2β3 · · · ∈ C. Ahora bien,
veamos que, en efecto, para todo n ∈ N, se tiene que γn = αn+βn2 . Para ello, sea n ∈ N y
consideremos tres posibles casos:
a. Si γn = 0 : Así, αn = 0, βn = 0 y por tanto, γn =
αn+βn
2
.
b. Si γn = 1 : Así, αn = 2, βn = 0 y por tanto,
γn = 1 =
2 + 0
2
=
αn + βn
2
=⇒ γn =
αn + βn
2
.
c. Si γn = 2 : Así, αn = 2, βn = 2 y por tanto,
γn = 2 =
2 + 2
2
=
αn + βn
2
=⇒ γn =
αn + βn
2
.
23
En cualquier caso, tenemos que, para todo n ∈ N, se tiene que γn = αn+βn2 y en consecuencia,
z
2
= x+y
2
, de donde �nalmente, se sigue que z = x+ y y x, y,∈ C.
Problema 22. Sea w ∈ [−1, 1] y hallemos x, y ∈ C tales que w = x + y. Como w ∈ [−1, 1],
entonces w + 1 ∈ [0, 2] y por el ejercicio anterior, tenemos que existe xy ∈ C tales que w + 1 =
x+ y, sin embargo, de esto se sigue que w = x− (1− y), no obstante, por el Problema 17, como
y ∈ C, entonces 1− y ∈ C. Finalmente, w = x− (1− y) y x, 1− y ∈ C.
Problema 23. Veamos que el conjunto de Cantor es no contbale. Para ello, análogo al razona-
miento del Problema 11 del Capítulo 1, veamos que ninguna enumeración contable de C logra
capturar todos sus elementos. Supongamos que C = {cn}n∈N, donde para todo n ∈ N, tenemos
que:
cn = .αn1αn2αn3 . . . ,
donde para todo i ∈ N, se tiene que αni ∈ {0, 2}. Notemos que no es difícil veri�car que las
expansiones ternarias que consisten solo de 0‘s y 2‘s son únicas. Ahora bien, consideremos la
sucesión de números:
βn :=

0 si αnn = 2.
2 si αnn = 0.
Y de�namos c := β1β2β3 . . . Es claro que c ∈ C porque su expansión decimal ternaria, por
de�nición, solo consiste de 0‘s y 2′s. Notemos además, que c di�ere de todos los elementos cni,
pues por lo menos di�eren en la n-ésima posición. Finalmente, hemos construido un elemento
de C que no es capturado por le enumeración contable; es decir, ninguna enumeración contable
logra capturar todos los elementos de C y, en consecuencia, C es no contable.
Observación 2. En el libro se demuestra que λ(C) = 0 y acabamos de demostrar que C es
no contable. De manera que el conjunto ternario de Cantor es un ejemplo de un conjunto no
contable cuya medida es cero. Por otro lado, si bien se ha demostrado que el conjunto de Cantor
es no contable, de ahí no se sigue que |C| = |R|, aunque resulta ser cierto, pues no es difícil
construir una función sobreyectiva entre [0, 1] y C.
Problema 24. Sean A,B ⊆ Rn tales que A,B ∈ L0 y �jemos � > 0. Por el teorema de
aproximación, tenemos que existen K1, K2 compactos y G1, G2 abiertos tales que:
K1 ⊆ A ⊆ G1, λ(G1 \K1) <
�
2
y K2 ⊆ A ⊆ G2, λ(G2 \K2) <
�
2
.
a. Unión: de�namos K := K1 ∪K2 y G := G1 ∪G2. Es claro que K es compacto, pues K1 y
K2 lo son. Y G es abierto, pues G1 y G2 lo son. Así, se tiene que:
K ⊆ A ∪B ⊆ G, y además,
G \K ⊆ (G1 \K1) ∪ (G2 \K2),
razón por la cual, se tiene que por la propiedad de medida para abiertos (O5),
λ(G \K) ≤ λ(G1 \K1) + λ(G2 \K2) <
�
2
+
�
2
= �.
Luego, K ⊆ A ∪ B ⊆ G y λ(G \ K) < �. De modo que, de nuevo por el teorema de
aproximación, tenemos que A ∪B ∈ L0.
24
b. Intersección: de�namos K := K1 ∩K2 y G := G1 ∩G2. Así, K es compacto, G es abierto,
K ⊆ A ∩B ⊆ G y G \K ⊆ (G1 \K1) ∪ (G2 \K2),
razón por la cual,
λ(G \K) ≤ λ(G1 \K1) + λ(G2 \K2) <
�
2
+
�
2
= �.
Así que,K ⊆ A∩B ⊆ G y λ(G\K) < �. De manera que, por la propiedad de aproximación,
tenemos �nalmente que A ∩B ∈ L0.
2.2. Propiedades de la medida de Lebesgue
Problema 25. Sea {Ak}k∈N una colección de subconjuntos medibles de Rn encajados, es decir,
tales que (∀k ∈ N)(Ak+1 ⊆ Ak). Y supongamos además que λ(A1) < ∞. Notemos que, dadas
las condiciones, podemos escribir a A1 como la siguiente unión disjunta:
A1 =
(
A1 ∩
∞⋂
k=2
Ak
)
∪
(
A1 \
∞⋂
k=2
Ak
)
.
Además, usando leyes de De Morgan, tenemos que:
A1 \
∞⋂
k=2
Ak = A1 ∩
(
∞⋂
k=2
Ak
)c
= A1 ∩
(
∞⋃
k=2
Ack
)
=
∞⋃
k=2
(Ack ∩ A1).
Ahora bien, como A1 lo expresamos como una unión disjunta, podemos calcular su medida en
virtud de la propiedad de aditividad contable (M4) como:
λ(A1) = λ
(
A1 ∩
∞⋂
k=2
Ak
)
+ λ
(
A1 \
∞⋂
k=2
Ak
)
= λ
(
∞⋂
k=2
Ak
)
+ λ
(
∞⋃
k=2
(Ack ∩ A1)
)
= λ
(
∞⋂
k=1
Ak
)
+ ĺım
n→∞
λ(Acn ∩ A1).
donde la tercera igualdad se da en virtud de la propiedad (M5), pues la colección {Acn∩A1}n∈N
satisface las hipótesis. Usando el hecho de que λ(A1) < ∞, podemos despejar en la última
igualdad para concluir que:
λ
(
∞⋂
k=1
Ak
)
= λ(A1)− ĺım
n→∞
λ(Acn ∩ A1)
= ĺım
n→∞
λ(An ∩ A1)
= ĺım
n→∞
λ(An)
25
Y como consecuencia, �nalmente, tenemos que:
λ
(
∞⋂
k=1
Ak
)
= ĺım
n→∞
λ(An).
Problema 26. Veamos que, en el ejercicio anterior, la hipótesis de λ(A1) < ∞ es necesaria.
Consideremos para k ∈ N �jo, los conjuntos los conjuntos Bkn de�nidos como sigue:
Bkn :=
(
k − 1
2n
, k +
1
2n
)
.
Y ahora de�nimos, para todo n ∈ N, los conjuntos An como sigue:
An :=
∞⋃
k=1
Bkn.
Notemos que, para cada k ∈ N �jo, los conjuntos Bkn son disjuntos y por tanto, podemos calcular
la medida de An en virtud de la propiedad de aditividad contable como:
λ(An) =
∞∑
k=1
λ(Bkn) =
∞∑
k=1
(
k +
1
2n
− k + 1
2n
)
=
∞∑
k=1
1
n
=∞.
En particular, λ(A1) =∞. Por otro lado, notemos que la colección {An}n∈N está encajada. En
efecto, sea x ∈ An+1, entonces:
k − 1
2n
< k − 1
2n+ 2
< x < k +
1
2n+ 2
< k +
1
2n
=⇒ x ∈ An.
Cuando n→∞, entonces An → {n} y por consiguiente,
λ
(
∞⋂
n=1
An
)
=
∞⋃
n=1
λ({n}) =
∞∑
n=1
0 = 0.
Finalmente,
λ
(
∞⋂
n=1
An
)
= 0 6=∞ = ĺım
n→∞
λ(An)
y en conclusión:
λ
(
∞⋂
n=1
An
)
6= ĺım
n→∞
λ(An).
Observación 3. En el ejercicio anterior usé el hecho de que la medida de un sigulete es cero y
si bien no se ha demostrado aún, no es difícil de demostrar. Sea A = {x} ⊆ R. Así que, comoA es �nito, entonces A es compacto. Sea � > 0 y de�namos:
I :=
[
x− �
4
, x+
�
4
]
.
Como A ⊆ I y ambos son compactos, tenemos que λ(A) ≤ λ(I) = x + �
4
− x + �
4
= �
2
< �.
Luego, para todo � > 0, se tiene que λ(A) ≤ 0 + � y por consiguiente, λ(A) = 0. De modo que,
la medida de un conjunto unipuntual es cero. Note que, en virtud de la propiedad de aditividad
contable, de acá se sigue inmediatamente que todo subconjunto contable de R tiene medida cero.
26
Problema 27. Si bien la medida exterior de un conjunto se de�nió usando conjuntos abierto,
se puede puede hacer directamente usando rectángulos especiales de la siguiente manera. Sea
A ⊆ Rn un conjunto arbitrario, entonces:
λ∗(A) = ı́nf
{
∞∑
k=1
λ(Ik) : A ⊆
∞⋃
k=1
Ik
}
,
donde, para cada k ∈ N, Ik es un rectángulo especial. Para demostrarlo, de�namos,
α := ı́nf
{
∞∑
k=1
λ(Ik) : A ⊆
∞⋃
k=1
Ik
}
,
y sea � > 0. Escojamos, para cada k ∈ N, I ′k como el interior de un rectángulo tal que:
Ik ⊆ I ′k y λ(I ′k) < λ(Ik) +
�
2k
.
Así, podemos calcular la medida exterior de A como:
λ∗(A) ≤ λ∗
(
∞⋃
k=1
I ′k
)
= λ
(
∞⋃
k=1
I ′k
)
≤
∞∑
k=1
λ(I ′k)
<
∞∑
k=1
[
λ(Ik) +
�
2k
]
=
∞∑
k=1
λ(Ik) + �
∞∑
k=1
1
2k
=
∞∑
k=1
λ(Ik) + �.
De donde se sigue que λ∗(A) ≤
∑∞
k=1 λ(Ik) y como
⋃∞
k=1 Ik se tomó arbitrariamente, entonces
λ∗(A) es una cota inferior del conjunto {
∑∞
k=1 λ(Ik) : A ⊆
⋃∞
k=1 Ik} , es decir, λ∗(A) ≤ α.
Por le otro lado, por de�nición de medida exterior, existe G ⊆ Rn abierto tal que:
A ⊆ G y λ∗(A) + � > λ(G).
Ahora bien, como G es abierto, por el Problema 10, se puede descomponer como una unión
contable de rectángulos especiales que no se traslapan, digamos,
G =
∞⋃
k=1
Ik.
Además, por el Problema 10, podemos calcular la medida de G como:
λ(G) = λ
(
∞⋃
k=1
Ik
)
=
∞∑
k=1
λ(Ik).
27
De manera que, tenemos:
α− � ≤
∞∑
k=1
λ(Ik)− � = λ(G)− � < λ∗(A),
de modo que, α − � < λ∗(A) y por tanto, concluimos que α ≤ λ∗(A). Finalmente, se sigue que
λ∗(A) = α.
Problema 28. Supongamos que A ∪B es medible y que:
λ(A ∪B) = λ∗(A) + λ∗(B) <∞.
Veamos que B es medible, el razonamiento para A es completamente análogo. Como A ⊆ A∪B
y este último es medible, podemos usar (M11) para calcular:
λ(A ∪B) = λ∗(A) + λ∗(B \ A).
De manera que, de las dos ecuaciones anteriores se obtiene que λ∗(B) = λ∗(B \ A), y por
consiguiente:
λ∗(B) = λ∗(B \ A) ≤ λ∗(B),
es decir, λ∗(B) ≤ λ∗(B) y por tanto, λ∗(B) = λ∗(B). Finalmente, como λ∗(B) < ∞, se sigue
que B es medible.
Problema 29. Supongamos que A y B son medibles. Así pues, tenemos que, por la propiedad
(M10):
λ∗(A) = λ∗(A) = λ(A) y λ
∗(B) = λ∗(B) = λ(B).
Por otro lado, como A ∩ B ⊆ A, A ∩ B ⊆ B, B \ (A ∩ B) = B \ A y A \ (A ∩ B) = A \ B,
entonces, en virtud de la propiedad (M11), tenemos que:
λ(A) = λ(A \B) + λ(A ∩B). (2.1)
λ(B) = λ(B \ A) + λ(A ∩B). (2.2)
De manera, que si sumamos (1) y (2), obtenemos:
λ(A) + λ(B) = λ(A ∩B) + λ(A ∩B) + λ(A \B) + λ(B \ A). (2.3)
Por otro lado, ya que podemos escribir a A∪B como una unión disjunta de la siguiente manera:
A ∪B = (A \B) ∪ (A ∩B) ∪ (B \ A),
podemos usar la propiedad de aditividad contable (M4) para calcular su medida como:
λ(A ∪B) = λ(A \B) + λ(A ∪B) + λ(B \ A). (2.4)
Y remplazando en (3), se sigue que, �nalmente:
λ(A) + λ(B) = λ(A ∪B) + λ(A ∩B).
O, equivalentemente:
λ(A ∪B) = λ(A) + λ(B)− λ(A ∩B).
28
Observación 4. Con este ejercicio, obtenemos una forma explícita de calcular la medida de
A ∪ B y A ∩ B partiendo de las medidas de A y de B. Sin embargo, aún no tenemos una
para calcular la medida de la resta A \B. Para hallarla, supongamos que A y B son conjuntos
medibles. Como A \B ⊆ A y A \ (A \B) = A∩B, entonces, por la propiedad (M11), tenemos
que:
λ(A \B) + λ(A ∩B) = λ(A),
de donde se sigue que:
λ(A \B) = λ(A)− λ(A ∩B).
Problema 30. Sean A,B ⊆ Rn.
a. Veamos que λ∗(A) + λ∗(B) ≥ λ∗(A ∪B) + λ∗(A ∩B).
Sea � > 0. Por aproximación a ín�mo, existen conjuntos abiertos G1 y G1 tales que:
A ⊆ G1 y λ(G1)−
�
2
< λ∗(A).
B ⊆ G2 y λ(G2)−
�
2
< λ∗(B).
De manera que,
λ∗(A ∪B) + λ∗(A ∩B)− � ≤ λ(G1 ∪G2) + λ(G1 ∩G2)− �
= λ(G1) + λ(G2)− �
< λ∗(A) + λ∗(B).
Y por tanto, como � se tomó de manera arbitraria, tenemos que
λ∗(A ∪B) + λ∗(A ∩B) ≤ λ∗(A) + λ∗(B).
b. Veamos que λ∗(A) + λ∗(B) ≤ λ∗(A ∪B) + λ∗(A ∩B).
Sea � > 0. Por aproximación, al supremo, existe conjuntos compactos K1, K2 tales que:
K1 ⊆ A y λ∗(A) < λ(K1) +
�
2
,
K2 ⊆ B y λ∗(B) < λ(K2) +
�
2
.
Así que, por el Problema 29, tenemos que:
λ∗(A) + λ∗(B) < λ(K1) + λ(K2) + �
= λ(K1 ∪K2) + λ(K1 ∩K2) + �
≤ λ∗(A ∪B) + λ∗(A ∩B) + �.
Y como � > 0 se escogió arbitrariamente, tenemos que:
λ∗(A) + λ∗(B) ≤ λ∗(A ∪B) + λ∗(A ∩B).
Observación 5. La igualdad en este ejercicio se da cuando A ⊆ B.
29
Problema 31. Empecemos por ver que si x = (x1, . . . , xn) ∈ Rn, entonces λ({x}) = 0. Sea
� > 0 y de�namos, para todo i ∈ Jn, el rectángulo especial:
I :=
[
x1 − n
√
�
2n+1
, x1 +
n
√
�
2n+1
]
× · · · ×
[
xn − n
√
�
2n+1
, xn +
n
√
�
2n+1
]
.
Es claro que x ∈ I y en consecuencia, {x} ⊆ I y como ambos son compactos, pues {x} es �nito,
se tiene que:
λ({x}) ≤ λ(I) =
(
2 n
√
�
2n+1
)n
= 2n
�
2n+1
=
�
2
< �.
De manera que, para todo � > 0, tenemos que λ{x} < 0 + �, y por tanto, λ({x}) = 0.
Ahora bien, sea A ⊆ Rn un conjunto contable. Así que, se puede pensar en A como la
colección A = {an : n ∈ N} y por tanto, podemos escribir a A como una unión disjunta de
conjuntos unipuntuales y disjuntos
A =
∞⋃
n=1
{an}.
De manera que, en virtud de la propiedad de aditividad contable (M4), tenemos que:
λ(A) = λ
(
∞⋃
n=1
{an}
)
=
∞∑
n=1
λ({xn}) =
∞∑
n=1
0 = 0.
De modo que, �nalmente, λ(A) = 0.
Observación 6. Así que todo conjunto contable tiene medida cero; sin embargo, el recíproco
no es cierto y un ejemplo es el conjunto ternario de Cantor.
Problema 32. Sea a ∈ R �jo. Veamos que λ({a} × Rn−1) = 0. Para ello, consideremos, para
todo k ∈ N, los rectángulos especiales:
Ak := {a} × [−k, k]n−1.
Notemos que, para cada k ∈ N, λ(Ak) = 0. Por otro lado, la sucesión {Ak}n∈N está encajada y
además, es claro que:
{a} × Rn =
∞⋃
k=1
Ak.
Así que, por la propiedad (M5), tenemos que:
λ({a} × Rn) = λ
(
∞⋃
k=1
Ak
)
= ĺım
k→∞
λ(Ak) = ĺım
k→∞
0 = 0.
Finalmente, λ({a} × Rn−1) = 0.
Problema 34. En general, no es cierto que si A ⊆ Rn es medible y su medida es positiva,
entonces A contiene un punto interior. Por ejemplo, R \ Q tiene medida in�nita y no contiene
puntos interiores. Sin embargo, esto no se debe al hecho de que el conjunto de los irracionales no
es cerrado. Para ver esto, construyamos A ⊆ [0, 1] cerrado, con medida positiva y cuyo interior
es vacío, usando un argumento similar la que se llevó a cabo en el Problema 12.
30
Sea � ∈ R tal que 0 < � < 1. Fijemos una numeración de los números racionales en el
intervalo [0, 1], {qk}k∈N. Y consideremos el conjunto abierto:
G :=
∞⋃
k=1
(
qk −
�
2k+1
, qk +
�
2k+1
)
.
Como G es abierto, es medible y:
λ(G) ≤
∞∑
k=1
λ
(
qk −
�
2k+1
, qk +
�
2k+1
)
=
∞∑
n=1
�
2k
= �.
Ahora bien, de�namos A := [0, 1] \G. Entonces, A es medible por ser la resta de dos conjuntos
medibles y por la Observación 4:
λ(A) = λ([0, 1])− λ([0, 1] ∩G) = λ([0, 1])− λ(G) = 1− λ(G) > 1− � > 0.
De manera que, A ⊆ [0, 1], es medible, con medida positiva y es cerrado por ser una intersección
�nita de cerrados. Así que, solo resta veri�car que A◦ = ∅. Para ello, sean x ∈ A y r > 0.
Entonces, existe qm ∈ Q, tal que qm ∈ (x − r, x + r). Y por tanto, como qm /∈ A, entonces
(x− r, x + r) no está contenido en A y por consiguiente, A no tiene puntos interiores, es decir
A◦ = ∅. Finalmente, A ⊆ [0, 1] tiene medida positiva, es cerrado y su interior es vacío.
Problema 35. ¾Existe A ⊆ [0, 1] cerrado tal que λ(A) = 1 y cuyo interior es vacío? Suponga-
mos que sí y sea A un conjunto con dichas características. Notemos que A es un subconjunto
propio de [0, 1], ya que este último satisface todas las condiciones, pero tiene interior no vacío.
Consideremos el conjunto G := (0, 1) \A. Notemos que G es abierto porque es una intersecciónde conjuntos abiertos, a saber Ac y (0, 1). Ahora bien, podemos calcular la medida de G como:
λ(G) = 1− λ([0, 1] \ (G)) ≤ 1− λ(A) = 0.
De modo que G tiene medida cero y en este caso, G = ∅. En consecuencia,
(0, 1) = (0, 1 ∩Gc) = (0, 1) ∩ A ∪ (0, 1)c = (0, 1) ∩ A,
y por tanto, A ⊆ (0, 1), razón por la cual, todo conjunto que satisfaga las condiciones descritas
al principio del ejercicio, es un subconjunto de (0, 1) y por tal razón, no puede tener interior
vacío. Finalmente, no existe un conjunto con dichas características.
Problema 37. Sea E ⊆ Rn tal que λ∗(E) < ∞. Por la propiedad de aproximación al ín�mo,
para todo k ∈ N, existe Gk ⊆ Rn abierto tal que:
E ⊆ Gk y λ(Gk) < λ∗(E) +
1
k
.
Ahora, de�namos:
A :=
∞⋂
k=1
Gk,
el cual es un conjunto medible, ya que es una intersección contable de conjuntos medibles.
Además, es claro que E ⊆ A y, por la selección de los abiertos, para todo k ∈ N:
λ(A) ≤ λ(Gk) < λ∗(E) +
1
k
,
con lo que, λ(A) ≤ λ∗(E). Por otro lado, como E ⊆ A, entonces λ∗(E) ≤ λ∗(A) = λ(A).
Finalmente, λ∗(E) = λ(A). Al conjunto A se le llama una envolvente medible de E.
31
Problema 38. Sean A,E ⊆ Rn tales que λ∗(E) <∞ y E ⊆ A ∈ L. Notemos que, como E ⊆ A
y A es medible, entonces en virtud de la propiedad (M11), podemos calcular la medida de A
como:
λ(A) = λ∗(E) + λ∗(A \ E).
De manera, que A es un envolvente medible de E si, y solo si, λ∗(E) = λ(A) y esto se da, si y
solo si, λ∗(A \ E) = 0.
Problema 41. Supongamos que {Ak}k∈N son medibles y que:
∞∑
k=1
λ(Ak) <∞.
Y por tanto, se tiene:
ĺım
n→∞
(
∞∑
k=n
λ(Ak)
)
= 0 y λ
(
∞⋃
k=1
λ(Ak)
)
≤
∞∑
k=1
λ(Ak) <∞.
Ahora bien, para todo n ∈ N, consideremos el conjunto Bn de�nido como:
Bn :=
∞⋃
k=n
Ak.
Notemos que, para cada n ∈ N, Bn es medible, ya que es una unión contable de conjuntos
medibles. Además, particularmente B1 tiene medida �nita, ya que:
λ(B1) = λ
(
∞⋃
k=n
Ak
)
≤
∞∑
k=1
λ(Ak) <∞.
De modo que, la colección {Bn}n∈N satisface las hipótesis de la propiedad (M6) y por consi-
guiente:
λ
(
ĺım sup
k→∞
Ak
)
= λ
(
∞⋂
n=1
(
∞⋃
k=n
Ak
))
= λ
(
∞⋂
n=1
Bn
)
= ĺım
n→∞
λ(Bn)
= ĺım
n→∞
(
∞∑
k=n
λ(Ak)
)
= 0.
De manera que, �nalmente,
λ
(
ĺım sup
k→∞
Ak
)
= 0.
32
Problema 43. Decimos que una colección de conjuntos medibles {Ak}k∈N es casi disjunta si
para todos i, j ∈ N, si i 6= J, entonces λ(Ai ∩ Aj) = 0.
Ahora veamos que las intersecciones de una colección casi disjunta no aportan valor a la
medida en uniones contables. Sea {Ak}k∈N un colección de conjuntos medibles casi disjunta.
Ya sabemos que podemos expresar a la unión de dicha familia, como una unión de conjuntos
disjuntos de la siguiente manera:
B1 := A1 y (∀k ∈ N)
(
Bk := Ak \
k−1⋃
i=1
Ak
)
.
Así pues,
∞⋃
k=1
Ak =
∞⋃
k=1
Bk,
y como dijimos antes, la segunda unión es disjunta. Por otro lado, para todo k ∈ N, podemos
expresar a An como una unión disjunta:
Ak = Bk ∪
(
Ak ∩
(
k−1⋃
i=1
Ai
))
,
y por la aditividad, podemos calcular la medida de Ak:
λ(Ak) = λ(Bk) + λ
(
Ak ∩
(
k−1⋃
i=1
Ai
))
,
no obstante, como la colección {Ak}k∈N es disjunta, tenemos que:
λ
(
Ak ∩
(
k−1⋃
i=1
Ai
))
≤
k−1∑
i=1
λ(Ak ∩ Ai) =
k−1∑
i=1
0 = 0,
razón por la cual, λ(Ak) = λ(Bk) para todo k ∈ N. Para concluir, teniendo en cuenta que la
unión de {Bk}k∈N es disjunta, podemos calcular:
λ
(
∞⋃
k=1
Ak
)
= λ
(
∞⋃
k=1
Bk
)
=
∞∑
k=1
λ(Ak) =
∞∑
k=1
λ(Bk).
Finalmente,
λ
(
∞⋃
k=1
Ak
)
=
∞∑
k=1
λ(Bk)
Problema 44. Supongamos que {Ak}k∈N es una colección de conjuntos medibles tales que
λ
(
∞⋃
k=1
Ak
)
=
∞∑
k=1
λ(Ak) <∞.
Para todo k ∈ N, de�namos los conjuntos Bk como en el problema anterior. Así, se tiene que:
∞∑
k=1
λ(Bk) = λ
(
∞⋃
k=1
Bk
)
= λ
(
∞⋃
k=1
Ak
)
=
∞∑
k=1
λ(Ak) <∞.
33
De manera, que si restamos la series, tenemos:
∞∑
k=1
λ(Bk)−
∞∑
k=1
λ(Ak) = 0 =⇒
∞∑
k=1
[λ(Ak)− λ(Bk)] = 0.
Y como para todo k ∈ N, λ(Ak)−λ(Bk) ≥ 0, se concluye que para todo k ∈ N, λ(Ak) = λ(Bk).
Por otro lado, al igual que en el ejercicio anterior, podemos calcular la medida de Ak como:
λ(Ak) = λ(Bk) + λ
(
Ak ∩
(
k−1⋃
i=1
Ai
))
y por tanto, para todo k ∈ N, se da que:
λ
(
Ak ∩
(
k−1⋃
i=1
Ai
))
= 0 =⇒
k−1∑
i=1
λ(Ak ∩ Ai) = 0.
De donde se sigue �nalmente que para todo k ∈ N, y todo i ∈ Jk, λ(Ak ∩ Ai) = 0, es decir, la
colección {Ak}k∈N es una colección casi disjunta.
34
Capítulo 3
Invarianza de la medida de Lebesgue
3.1. Un poco de álgebra lineal
Problema 1. Sea F : Rn → Rn una función lineal. Veamos que existe una única matriz n× n
tal que para todo x ∈ Rn, F (x) = Tx. En efecto, consideremos la base canónica de Rn, denotada
por: e1, e2, . . . , en.
a. Existencia: consideremos la matriz de�nida por columnas como:
T =
(
F ((e1) · · · F (en)
)
=
(
T1 · · · Tn
)
Es decir, la k-ésima columna de T es el resultado de aplicarle F al vector canónico ek. Sea
x = (x1, . . . , xn) ∈ Rn. Entonces,
F (x) = F (x1e1 + · · ·+ xnen)
= x1F (e1) + · · ·+ xnF (en)
= Tx.
Luego, para todo x ∈ Rn, se tiene que F (x) = Tx.
b. Unicidad: sea T ′ una matriz n× n tal que para todo x ∈ Rn se tiene que T ′(x) = F (x) y
supongamos que:
T ′ =
(
T ′1 · · · T ′n
)
Ya que para todo vector x ∈ Rn se tiene que T ′(x) = F (x), entonces, para todo k ∈ Jn,
tenemos que:
Tk = F (ek)
= T ′ek
= 0T ′1 + 0T
′
2 + · · ·+ T ′k + 0T ′k+1 + · · ·+ 0T ′n
= T ′k.
De manera que, para todo k ∈ Jn, tenemos que Tk = T ′k y en consecuencia, T = T ′.
Finalmente, existe una única matriz T tal que F (x) = Tx.
Problema 2. Sea F : Rn → Rm una función lineal. Veamos que existe una única matriz m×n
tal que para todo x ∈ Rn, F (x) = Tx. En efecto, consideremos la base canónica de Rn, denotada
por: e1, e2, . . . , en.
35
a. Existencia: consideremos la matriz de�nida por columnas como:
T =
(
F ((e1) · · · F (en)
)
=
(
T1 · · · Tn
)
Es decir, la k-ésima columna de T es el resultado de aplicarle F al vector canónico ek.
Como el codominio de la función F es Rm, tenemos que T es una matriz m× n. Fijemos
x = (x1, . . . , xn) ∈ Rn. Entonces,
F (x) = F (x1e1 + · · ·+ xnen)
= x1F (e1) + · · ·+ xnF (en)
= Tx.
Luego, para todo x ∈ Rn, se tiene que F (x) = Tx.
b. Unicidad: sea T ′ una matriz m× n tal que para todo x ∈ Rn se tiene que T ′(x) = F (x) y
supongamos que:
T ′ =
(
T ′1 · · · T ′n
)
Ya que para todo vector x ∈ Rn se tiene que T ′(x) = F (x), entonces, para todo k ∈ Jn,
tenemos que:
Tk = F (ek)
= T ′ek
= 0T ′1 + 0T
′
2 + · · ·+ T ′k + 0T ′k+1 + · · ·+ 0T ′n
= T ′k.
De manera que, para todo k ∈ Jn, tenemos que Tk = T ′k y en consecuencia, T = T ′.
Finalmente, existe una única matriz T tal que F (x) = Tx.
Problema 3. Decimos que una función F que mapea un espacio vectorial en otro es una
función afín si para todo x, y ∈ Rn y todo a ∈ R, se tiene que:
F (ax+ (1− a)y) = aF (x) + (1− a)F (y).
Veamos que F es una función afín si, y solo si, existen x0 �jo y una función lineal F0 tales que
para todo x ∈ Rn:
F (x) = x0 + F0(x).
En efecto, supongamos que F existen x0 y una función lineal F0 tales que para todo x ∈
Rn, F (x) = x0 + F0(x) y sean x, y ∈ Rn y a ∈ R. Entonces:
F (ax+ (1− a)y) = x0 + F0(ax+ (1− a)y)
= x0 + aF0(x) + (1− a)F0(y)
= a(x0 + F0(x)) + (1− a)(x0 + F0(y))
= aF (x) + (1− a)F (y).
Y por consiguiente, F es una función afín.
36
Recíprocamente, supongamos que F es afín. De�namos x0 := F (0), y naturalmente, como
queremos que F (x) = x0 + F0(x), consideremos F0(x) := F (x) − x0. Veamos que F0 es una
función lineal. sean a, b ∈ Rn tales que a 6= −b y calculemos:
F0(ax+ by) = F (ax+ by)− x0
=
a
a+ b
F ((a+ b)x) +
b
a+ b
F ((a+ b)y)− x0
=
a
a+ b
((a+ b)F (x) + 1− (a+ b)x0) +
b
a+ b
((a+ b)F (y) + (1− (a+ b)x0))− x0
= aF (x)− ax0 + bF (y)− bx0
= a[F (x)− x0] + b[F (y)− x0]
= aF0(x) + bF0(y).
De manera similar se sigue si a = −b. Así que, F0 es efectivamente una función lineal.
Finalmente, veamos que x0 y F0 son únicos y están determinados por F. Sean x′0 �jo y F
′
0
una función lineal tales que F (x) = x′0 +F
′
0(x). Notemos que F (0) = x
′
0 +F
′
0(0), pero, al ser F
′
0
lineal, tenemos que:
F ′0(0) = F
′
0(x− x)= F ′0(x)− F ′0(x) = 0.
De modo que, x′0 = F (0) = x0, de donde se sigue que x
′
0 = x0 y F
′
0 = F0. Finalmente, x0 y F0
son únicas y están determinadas por F.
Problema 4. Sean F y G funciones lineales y digamos que M es la matriz de la función
composición F ◦ G. Por el Problema 1, sabemos que M está determinada por su acción en la
base canónica de Rn de la siguiente manera:
M = ((F ◦G)e1 · · · (F ◦G)en)
= (F (G(e1)) · · ·F (G(en)))
= (F (Te1) · · ·F (Ten))
= (STe1 · · ·STen)
= ST.
Problema 5. De�namos Z := {T ∈ Mn×n(R) : (∀S ∈ Mn×n(R))(ST = TS)}, es decir, Z es
el centro de las matrices. Sea T ∈ Z y veamos que existe c ∈ R tal que T = cId. Comparemos
las entradas de la diagonal principal de los productos ST y TS. Para ello, para todo k ∈ Jn,
de�namos las siguientes matrices por columnas:
M1 :=
(
e1 0 · · · 0
)
=

1 0 · · · 0
0 0 · · · 0
...
. . . . . . 0
0 0 · · · 0

M2 :=
(
0 e2 · · · 0
)
=

0 0 · · · 0
0 1 · · · 0
...
. . . . . . 0
0 0 · · · 0

37
Mn :=
(
0 0 · · · en
)
=

0 0 · · · 0
0 0 · · · 0
...
. . . . . . 0
0 0 · · · 1

Notemos que si TMk = MkT, entonces la entrada en la �la k y la columna k de la matriz
S es el único elemento distinto de cero. Luego, si para todo k ∈ Jn se tiene que TMk = MkT,
entonces T es una matriz diagonal. Ahora bien, si intercambiamos las columnas de la matriz
identidad, y multiplicamos por T, obtenemos que todos las entradas de la diagonal de T son
iguales y por tanto, existe c ∈ R tal que T = cId. Notemos que, es claro que si T tiene la
forma cId para algún c ∈ R, entonces T conmuta con todas las matrices. Luego, el centro de las
matrices n× n con entradas reales es:
Z(Mn×n(R)) = {cId : c ∈ R} .
Problema 6. Sea T una matriz. Veamos que T es ortogonal si, y solo si, la función lineal
asociada a T preserva el producto interno de Rn, es decir, si para todo x, y ∈ Rn, se tiene que:
Tx · Tx = x · y.
En efecto, por un lado, supongamos que T es ortogonal y sean x, y ∈ Rn. Entonces,
Tx · Ty = xtrT tr · Ty
= xtr · y
= x · y.
Luego, Tx efectivamente, preserva el producto interno.
Recíprocamente, supongamos que para todo x, y ∈ Rn se tiene que Tx · Ty = x · y. Así que,
para i, j ∈ Jn se da que:
Tei · Tej = ei · ej =
{
1 si i = j
0 si i 6= j
De manera que,
T trT =
(Te1)
tr
...
(Ten)
tr
(Te1 · · ·Ten) = Id.
Y por tanto, T = T tr.
Finalmente, T es ortogonal si, y solo si, la función lineal asociada a T preserva el producto
interno de Rn.
Problema 7. Sean T una matriz, M una matriz elemental de multiplicación y A una matriz
elemental de adición. Veamos qué consecuencias tiene multiplicar a T por izquierda por estas
matrices elementales:
a. Multiplicación: TM es la matriz que se obtiene desde T al multiplicar la columna k por
el número c. Veamos un ejemplo.
TM =
a b cd e f
g h i
2 0 00 1 0
0 0 1
 =
2a b c2d e f
2g h i

38
En este caso, se multiplica por c = 2 la columna 1. Recordemos que si se multiplica por
derecha, entonces se obtiene el mismo resultado, pero el �la y no en la columna.
b. Adición: TA es la matriz que se obtiene desde T al sumar c veces la columna l a la columna
j. Veamos un ejemplo.
TM =
a b cd e f
g h i
1 2 00 1 0
0 0 1
 =
a 2a+ b cd 2d+ e f
g 2g + h i

En este caso, se multiplica por c = 2 la columna 1. Recordemos que si se multiplica por
derecha, entonces se obtiene el mismo resultado, pero el �la y no en la columna.
Problema 8. Se puede de�nir un tercer tipo de matrices elementales. Supongamos que k 6=
l. Entonces S es una matriz de transposición si ST es la matriz que se obtiene de T por
intercambiar las �las k y l de T. Según esta descripción, de�namos cada entrada de S como
sigue:
sii := 1, si i 6= k o i 6= l.
skl = slk := 1.
sij := 0, para el resto de parejas (i, j).
Problema 11. Consideremos A como una matriz 2× 2 invertible:
A =
(
a b
c d
)
.
Siguiendo la construcción del lema, consideremos dos casos:
a. a 6= 0:
A =
(
a 0
0 1
)(
1 0
c 1
)(
1 0
0 ad−bc
a
)(
1 b
a
0 1
)
.
Veri�quemos que en efecto es así:(
a 0
c 1
)(
1 0
c 1
)(
1 0
0 ad−bc
a
)(
1 b
a
0 1
)
=
(
a 0
c 1
)(
1 0
0 ad−bc
a
)(
1 b
a
0 1
)
=
(
a 0
c ad−bc
a
)(
1 b
a
0 1
)
=
(
a b
c d
)
= A.
b. a = 0: en este caso, como A es invertible, entonces c 6= 0.
A =
(
1 −1
c
0 1
)(
1 0
c 1
)(
1 0
0 −bc
)(
1 b+ d
c
0 1
)
.
39
Veri�quemos que en efecto es así:(
1 −1
c
0 1
)(
1 0
c 1
)(
1 0
0 −bc
)(
1 bd
c
0 1
)
=
(
0 −1
c
c 1
)(
1 0
0 −bc
)(
1 b+ d
c
0 1
)
=
(
0 b
c −bc
)(
1 b+ d
a
0 1
)
=
(
0 b
c d
)
= A.
3.2. Matrices ortogonales
Problema 15. Veamos que la colección de los movimientos rígidos de Rn, dotado con la ope-
ración composición, forman un grupo. Este grupo es llamado el grupo Euclideano.
Sean φ, ψ y θ movimientos rígidos de Rn. Así que, existen z1, z2, z3 ∈ Rn y matrices ortogo-
nales T1, T2 y T3 tales que para todo x ∈ Rn:
φ(x) = z1 + T1x y ψ(x) = z2 + T2x y θ(x) = z3 + T3x.
a. Operación binaria: veamos que φ ◦ ψ es un movimiento rígido.
(φ ◦ ψ)(x) = φ(ψ(x))
= φ(z2 + T2x)
= z1 + T1(z2 + T2x)
= z1 + T1z2 + T1T2x
= z + Tx.
Donde z := z1 + T1z2 y T := T1T2 y es claro que es ortogonal ya que:
TT tr = (T1T2)(T1T2)
tr
= (T1T2)(T
tr
2 T
tr
1 )
= T1T2T
tr
2 T
tr
1
= T1T
tr
1
= Id.
b. Asociatividad:
[(φ ◦ ψ) ◦ θ](x) = z1 + T1z2 + T1T3(z3 + T3x)
= z1 + T1(z2 + T3(z3 + T3x))
= [φ ◦ (ψ ◦ θ)](x).
Luego, (φ ◦ ψ) ◦ θ = φ ◦ (ψ ◦ θ).
c. Identidad: sea Id el movimiento rígido identidad.
(φ ◦ Id)(x) = φ(Id(x)) = z1 + T1x = φ(x).
(Id ◦ φ)(x) = Id(φ(x)) = Id(z1 + T1x) = z1 + T1x = φ(x).
Luego, φ ◦ Id = Id ◦ φ = φ.
40
d. Existencia de inversos: consideremos z2 := −T tr1 z1 y T2 := T tr1 y el movimiento rígido
de�nido por ψ(x) := z2 + T2x. Es claro que ψ es un movimiento rígido, ya que T2 es una
matriz ortogonal. Además, tenemos que:
(φ ◦ ψ)(x) = φ(ψ(x))
= φ(−T tr1 z1 + T tr1 x)
= z1 + T1(−T tr1 z1 + T tr1 x)
= z1 − T1T tr1 z1 + T1T tr1 x
= z1 − z1 + x
= x.
(ψ ◦ φ)(x) = ψ(φ(x))
= ψ(z1 + T1x)
= −T tr1 z1 + T tr1 (z1 + Tx)
= −T tr1 z1 + T tr1 z1 + T tr1 T1x
= x.
Luego, φ ◦ ψ = ψ ◦ φ = Id.
Problema 16. Sean x, y, z ∈ Rn tales que:
|x− y| = |x− z|+ |z − y|.
De�namos a := x− z y b := z− y. Entonces a+ b = x− y, y por consiguiente, |a+ b| = |a|+ |b|,
es decir, a y b satisfacen la igualdad en la desiguladad triangular y por tanto, por el Problema
13 del Capítulo 1, existe r > 0 tal que a = rb, es decir:
a = rb =⇒ x− z = r(z − y)
=⇒ x− ry = rz + z
=⇒ x− ry = z(r + 1)
=⇒ z =
(
1
r + 1
)
x−
(
r
r + 1
)
y.
Según lo que necesitamos, de�namos t ∈ [0, 1] como sigue:
t :=
1
r + 1
.
Notemos que:
1− t = 1− 1
r + 1
=
r + 1− 1
r + 1
=
r
r + 1
,
y por tanto, �nalmente, tenemos que:
z = tx− (1− t)y y t ∈ [0, 1].
41
Problema 17. Decimos que φ : Rn → Rn es una isometría , si para todo x, y ∈ Rn se tiene
que:
|φ(x)− φ(y)| = |x− y|.
Veamos que toda isometría φ : Rn → Rn es un movimiento rígido. Empecemos por notar que,
sin pérdida de generalidad, podemos suponer que φ(0) = 0, ya que si φ(0) 6= 0, podemos de�nir
φ′(x) := φ(x)− φ(0). Así que,
|φ′(x)− φ′(y)| = |φ(x)− φ(0)− φ(y) + φ(0)| = |φ(x)− φ(y)| = |x− y|.
De modo que, si φ es una isometría, entonces φ′ también lo es y además, φ′(0) = 0.
Ahora bien, sean x, y ∈ Rn y s ∈ R. Consideremos tres posibles casos:
a. s ∈ [0, 1]: así, tenemos que:
|φ(x)− φ(y)| = |x− y|
= s|x− y|+ (1− s)|x− y|
= |sx− sy + x− x|+ (1− s)|x− y|
= |x− sy − (1− s)x|+ |(1− s)x+ sy − y|
= |φ(x)− φ((1− s)x+ sy)|+ |φ((1− s)x+ sy)− φ(y))|.
De modo que, por el ejercicio anterior, existe t ∈ [0, 1] tal que:
φ((1− s)x+ sy) = (1− t)φ(x) + tφ(y)
φ((1− s)x+ sy) = φ(x)− tφ(x) + φ(y)
φ((1− s)x+ sy)− φ(x) = tφ(x)− tφ(y)
|φ((1− s)x+ sy)− φ(x)| = |tφ(x)− tφ(y)|
|(1− s)x+ sy − x| = t|y − x|
|s(x− y)| = t|y − x|
s|x− y| = t|y − x|
s = t.
Y por tanto, φ((1− s)x+ sy) = (1− s)φ(x) + sφ(y). Por otro lado, no es difícil demostrar
que si α > 0, entonces para todo x ∈ Rn, se tiene que φ(αx)αφ(x) y φ(−x) = −φ(x).
Usemos esto en los siguientes casos:
b. s > 1:
φ((1− s)x+ sy)= φ
(
1
s
(1− s)sx+ s− 1
s
s2
s− 1
y
)
=
1
s
φ((1− s)sx) + s− 1
s
φ
(
s2
s− 1
y
)
= φ((1− s)x) + sφ(y)
= (s− 1)φ(x) + sφ(y)
= (1− s)φ(x) + sφ(y).
Luego, φ((1− s)x+ sy) = (1− s)φ(x) + sφ(y).
42
c. s < 0:
φ((1− s)x+ sy) = φ
(
1
2
2(1− s)x+ 1
2
2sy
)
=
1
2
φ(2(1− s)x) + 1
2
φ(2sy)
= (1− s)φ(x)− sφ(−y)
= (1− s)φ(x) + sφ(y).
Luego, φ((1− s)x+ sy) = (1− s)φ(x) + sφ(y).
Finalmente, en cualquier caso, φ es una función afín, y por tanto, por el Problema 3, φ es un
movimiento rígido de Rn.
3.3. Matrices generales
Problema 18. Sea φ una isometría de Rn, es decir, que preserva las distancias y sin pérdida
de generalidad, supongamos que φ(0) = 0. Empecemos por notar que φ preserva las normas de
los vectores:
|φ(x)| = |φ(x)− 0| = |φ(x)− φ(0)| = |x− 0| = |x|.
Ahora bien, por las propiedades del producto interno, tenemos que:
|φ(x)− φ(y)|2 = (φ(x)− φ(y)) · (φ(x)− φ(y))
= φ(x) · φ(x)− 2φ(x) · φ(y) + φ(y) · φ(y)
= |φ(x)|2 − 2φ(x) · φ(y) + |φ(y)|2.
De manera completamente análoga, podemos escribir a |x− y|2 como:
|x− y|2 = |x|2 − 2x · y + |y|2.
Ahora bien, como φ es una isometría, y ya vimos que preserva las normas, tenemos que:
|φ(x)|2 − 2φ(x) · φ(y) + |φ(y)|2 = |φ(x)− φ(y)|2
= |x− y|
= |x|2 − 2x · y + |y|2.
(3.1)
De donde se sigue que:
φ(x) · φ(y) = x · y.
Y en consecuencia, toda isometría preserva el producto punto.
Ahora bien, �jemos �1, . . . , �n como una base ortonormal de Rn. Así que, para todo i ∈
Jn, |�i| = 1 y son ortogonales a pares. Así que, como φ preserva la norma y el producto interno,
entonces φ(�1), · · · , φ(�n) también es una base ortonormal de Rn y en consecuencia, todo vector
de Rn se puede escribir como una combinación lineal de ellos. Entonces, si de�nimos la matriz,
T :=
(
φ(�1) · · ·φ(�n)
)
T resulta ser una matriz ortogonal. Finalmente, tenemos que:
φ(x) = TT trφ(x) =
n∑
k=1
φ(�k)φ(�k)
trφ(x) =
n∑
k=1
φ(�k)(�k · x) =
n∑
k=1
φ(�k) · xk = Tx.
43
Problema 19. Sean a1, a2, . . . , an ∈ Rn y para cada j ∈ Jn, escribamos aj en componentes
como: aj := (a1j, a2j, . . . , aanj). Ahora bien, el �paralelogramo� determinado por a1, a2, . . . , an
se de�ne como:
A := {t1a1 + · · ·+ tnan : (∀k ∈ Jn)(0 ≤ tk ≤ 1)}.
Y consideremos la matriz M de�nida como:
M := (aij) =

a11 a12 · · · a1n
a21 a22 · · · a2n
...
. . . . . .
...
an1 an2 · · · ann

Veamos que λ(A) = | det(M)|. Para ello, de�namos la función T : Rn → Rn tal que T (x) := Mx,
es decir, la función lineal asociada a la matrizM. Y consideremos además, el n-cubo C := [0, 1]n.
Así, es claro que A = TC y por tanto, tenemos que:
λ(A) = | det(M)|λ(C)|,
sin embargo, λ(C) = 1 y por consiguiente,
λ(A) = | det(M)|.
44
Capítulo 4
Algunos conjuntos interesantes
4.1. Un conjunto no medible
Problema 0. Si bien el autor no plantea esto como un problema, es un ejercicio que deja
al lector y que juega un papel fundamental en la existencia de conjuntos no medibles. Sean
x, x′ ∈ Rn y veamos que x + Qn = x′ + Qn o bien (x + Qn) ∩ (x′ + Qn) = ∅. Para ello,
consideremos dos posibles casos:
a. x− x′ ∈ Qn: en este caso se da que x+Qn = x′+Qn. En efecto, sea y ∈ x+Qn. Así que,
existe q ∈ Qn tal que y = x+ q y por tanto:
y = x+ q + (x′ − x′) = x′ + (x− x′ + q) = x+ q1,
donde q1 := x− x′ + q ∈ Qn. Luego, y ∈ x′ + Qn y por consiguiente, x+ Qn ⊆ x′ + Qn.
De manera completamente análoga, se sigue que x′ + x ⊆ Qn. De manera que, en conse-
cuencia, x+ Qn = x′ + Qn.
b. x− x′ /∈ Qn: en este caso se da que (x+Qn)∩ (x′ + Qn) = ∅. En efecto, supongamos que
(x + Qn) ∩ (x′ + Qn) 6= ∅. Así que, existe y ∈ Rn tal que y ∈ x + Qn y y ∈ x′ + Qn. De
manera que, existe q1, q1 ∈ Qn tales que:
y = x+ q1 y y = x
′ + q2,
de donde x+q1 = x′+q2 y por tanto, x−x′ = q1−q2 ∈ Qn, lo cual es absurdo. Finalmente,
(x+Qn) ∩ (x′ + Qn) = ∅.
Observación 7. Aunque el libro opta por usar el hecho anterior en la construcción del conjunto
no medible E, esto se puede hacer más naturalmente usando clases de equivalencia de la siguiente
manera. De�namos sobre Rn la relación de equivalencia ∼ tal que x ∼ y si, y solo si, x−y ∈ Qn.
Es inmediato que ∼ es re�exiva y simétrica. Si x − y ∈ Qn y y − z ∈ Qn, entonces existen
q1, q2 ∈ Qn tales que x − y = q1 y y − z = q2. De manera que, x − z = q1 + q2 ∈ Qn y por
tanto, si x ∼ y y y ∼ z, entonces x ∼ z. De modo que ∼ es una relación de equivalencia sobre
Rn y por tanto, lo divide en clases de equivalencia, que además son disjuntas. Finalmente, es
su�ciente con que de�namos E como la colección de representantes de clase de las clases de
equivalencia, Rn/ ∼ .
45
Problema 1. Sea A ⊆ Rn un conjunto medible y acotado tal que λ(A) > 0. Por el corolario de
la sección 4.A, existe E ⊆ A tal que E es no medible, es decir, λ∗(E) < λ∗(E). Consideremos
B := A \E. Así, B ⊆ A y como A = E ∪B es medible, por la propiedad (M11), tenemos que:
λ∗(E) + λ∗(B) = λ∗(A).
De modo que, como E es no medible y A = E ∪B.
λ∗(E) + λ∗(B) < λ
∗(E) + λ∗(B)
= λ(A)
= λ(E ∪B)
= λ∗(E ∪B).
Y por tanto,
λ∗(E) + λ∗(B) < λ∗(E ∪B).
Ahora bien, razonando por contradicción, supongamos que λ∗(E) + λ∗(B) ≥ λ∗(E ∪B). Como
por el Problema 30 del Capítulo 2, se tiene que:
λ∗(E) + λ∗(B) ≤ λ∗(E ∪B) + λ∗(E ∩B) = λ∗(E ∪B),
entonces tendríamos que:
λ∗(E) + λ∗(B) = λ∗(E ∪B),
y en consecuencia, como E ∪ B es medible y tiene medida �nita, (pues A es acotado), por el
Problema 28 del Capítulo 2, tenemos que E y B son medibles, lo cual es absurdo. Así, se sigue
que:
λ∗(E) + λ∗(B) > λ∗(E ∪B).
Finalmente, E y B son conjuntos disjuntos y satisfacen que:
λ∗(E) + λ∗(B) > λ∗(E ∪B) y λ∗(E) + λ∗(B) < λ∗(E ∪B).
Problema 2. Sean A,B ⊆ Rn separados ; es decir, que existe C ⊆ Rn medible tal que:
A ⊆ C y λ(C ∩B) = 0.
Veamos que λ∗(A ∪ B) = λ∗(A) + λ∗(B). Para ello, apliquemos el criterio de Carathéodory
(M12) a B. Así pues, como C es medible, podemos calcular:
λ∗(B) = λ∗(B ∩ C) + λ∗(B ∩ Cc) = λ∗(B ∩ Cc),
y por tanto, λ∗(B) = λ∗(B ∩Cc). Por otro lado, aplicando de nuevo el criterio de Carathéodory
al conjunto A ∪B, tenemos que:
λ∗(A ∪B) = λ∗((A ∪B) ∩ C) + λ∗((A ∪B) ∩ Cc)
= λ∗((A ∪B) ∩ C) + λ∗(B ∩ Cc)
= λ∗((A ∪B) ∩ C) + λ∗(B).
46
De manera que, debemos analizar la medida exterior de (A∪B)∩C. Por un lado, es claro que
A ⊆ (A ∪B) ∩ C y entonces λ∗(A) ≤ λ∗((A ∪B) ∩ C). Por el otro lado, se tiene:
λ∗(A) = λ∗(A) + λ∗(B ∩ C)
≤ λ∗((A ∪B) ∩ C) + λ∗(A ∩B ∩ C)
≥ λ∗((A ∪B) ∩ C).
De donde se sigue que λ∗(A) ≥ λ∗((A ∪ B) ∩ C) y en consecuencia, λ∗(A) = λ∗((A ∪ B) ∩ C).
Finalmente, se tiene que:
λ∗(A ∪B) = λ∗(A) + λ∗(B).
Observación 8. Una consecuencia del ejercicio anterior es la siguiente: si A,B ⊆ Rn son
separados, A ∪B es medible y su medida es �nita, entonces A y B son medibles.
4.2. La función de Lebesgue asociada con un conjunto de
Cantor
Problema 7. Sean E ⊆ Rn y f : E → Rn una función uniformemente continua. Denotemos
por E ′ a la colección de puntos de acumulación de E. Recordemos que si x ∈ E ′, entonces existe
una sucesión de puntos {xk}k∈N tales que para todo k ∈ N, xk ∈ E y además:
x = ĺım
k→∞
xk.
Ahora bien, de�namos la función F : E → Rn como sigue:
F (x) :=

f(x), si x ∈ E
ĺımk→∞ f(xk), si x ∈ E ′
Es claro que F es una extensión de f ; es decir, en E coincide con f y no es difícil veri�car que,
cuando x ∈ E ′, entonces F (x) no depende de la elección de la sucesión que se haga para x.
Además, si x ∈ E y x es punto de acumulación de E, entonces como f es en particular continua,
es secuencialmente continua y por tanto, se tiene que:
f(x) = ĺım
k→∞
f(xk) = F (x),
es decir, cuando x ∈ E ∩ E ′, no hay ambigüedad en la de�nición de F (x). Ahora bien, veamos
que F es continua. Sea � > 0. Por la continuidad uniforme de f, existe δ > 0 tal que para todo
x, y ∈ E, si |x − y| < δ, entones |f(x) − f(y)| < �/2. De�namos δ′ := δ
3
> 0 y sean x, y ∈ E
tales que |x− y| < δ′. Consideremos tres posibles casos:
a. x, y ∈ E: en este caso, tenemos que como |x− y| < δ′ y δ′ < δ y por tanto,
|F (x)− F (y)| = |f(x)− f(y)| < �
2
< �
y por tanto, F es continua.
47
b. x ∈ E y x ∈ E ′: en este caso, como y es un punto de acumulación

Continuar navegando

Materiales relacionados

557 pag.
Solucionario_Fisica_I_y_II_Leyva

User badge image

Christian Enrique Estrella Osorio

255 pag.
Solucionario_Demidovich_TOMO_III_3

Colégio Bernoulli

User badge image

Estudante PD

20 pag.
Solucionario_Analisis_Real_Elon_Lages_Li

User badge image

Snaider seminario Llanfranco llacsahuanga